2010 AMC 10/12 A Math Jam
Go back to the Math Jam ArchiveAoPS Instructors lead a discussion of the harder problems and requested problems from both the 2010 AMC 10 A and the 2010 AMC 12 A.
Copyright © 2025 AoPS Incorporated. This page is copyrighted material. You can view and print this page for your own use, but you cannot share the contents of this file with others.
Facilitator: Richard Rusczyk
rrusczyk
2010-02-10 19:02:18
Welcome to the 2010 AMC 10A/12A Math Jam!
Welcome to the 2010 AMC 10A/12A Math Jam!
rrusczyk
2010-02-10 19:02:23
Before we get started I would like to take a moment to explain our virtual classroom to those who have not previously participated in a Math Jam or one of our online classes.
Before we get started I would like to take a moment to explain our virtual classroom to those who have not previously participated in a Math Jam or one of our online classes.
rrusczyk
2010-02-10 19:02:30
The classroom is moderated, meaning that students can type into the classroom, but these comments will not go directly into the room. These comments go to the instructors, who may choose to share your comments with the room.
The classroom is moderated, meaning that students can type into the classroom, but these comments will not go directly into the room. These comments go to the instructors, who may choose to share your comments with the room.
rrusczyk
2010-02-10 19:02:52
This helps keep the class organized and on track. This also means that only well-written comments will be dropped into the classroom, so please take time writing responses that are complete and easy to read
This helps keep the class organized and on track. This also means that only well-written comments will be dropped into the classroom, so please take time writing responses that are complete and easy to read
rrusczyk
2010-02-10 19:03:06
There are a lot of students here! As I said, only well-written comments will be passed to the entire group. Please do not take it personally if your comments do not get posted, and please do not complain about it. I expect this Math Jam to be much larger than our typical class, so please be patient with me---there are quite a few of you here tonight!!
There are a lot of students here! As I said, only well-written comments will be passed to the entire group. Please do not take it personally if your comments do not get posted, and please do not complain about it. I expect this Math Jam to be much larger than our typical class, so please be patient with me---there are quite a few of you here tonight!!
rrusczyk
2010-02-10 19:03:31
Also, we won't be going through the math quite as thoroughly as we do in our classes -- I can't teach all the prerequisite material for every problem as we go (and certainly not to 200 students)! Another difference between tonight and our regular online classes is that it is very unlikely that we'll be able to answer every single question you ask. We usually do in our classes, but we have a large number of students tonight! So, please go ahead and ask questions, but also please understand if we aren't able to answer them all!
Also, we won't be going through the math quite as thoroughly as we do in our classes -- I can't teach all the prerequisite material for every problem as we go (and certainly not to 200 students)! Another difference between tonight and our regular online classes is that it is very unlikely that we'll be able to answer every single question you ask. We usually do in our classes, but we have a large number of students tonight! So, please go ahead and ask questions, but also please understand if we aren't able to answer them all!
rrusczyk
2010-02-10 19:03:57
We do have an assistant tonight who can help answer some of the questions. Her name is Wendy Hou (wobster109). She is a college sophomore majoring in computer programming. She has taken the USAMO four times and competed in a broad spectrum of high school competitions, and she had the honor of representing the U.S. in China as a member of the first U.S.A. CGMO team. She has extensive experience from summer programs and with Art of Problem Solving, both as a student and as a mentor. She spends her days frantically battling down a monstrous courseload. She haunts a bell tower. At night, she transforms into a velociraptress and howls at the moon.
We do have an assistant tonight who can help answer some of the questions. Her name is Wendy Hou (wobster109). She is a college sophomore majoring in computer programming. She has taken the USAMO four times and competed in a broad spectrum of high school competitions, and she had the honor of representing the U.S. in China as a member of the first U.S.A. CGMO team. She has extensive experience from summer programs and with Art of Problem Solving, both as a student and as a mentor. She spends her days frantically battling down a monstrous courseload. She haunts a bell tower. At night, she transforms into a velociraptress and howls at the moon.
rrusczyk
2010-02-10 19:04:08
She'll be joining us in about a half hour.
She'll be joining us in about a half hour.
rrusczyk
2010-02-10 19:04:29
In her place, we have a super-sub, one of our instructors here at AoPS, Jeremy Copeland.
In her place, we have a super-sub, one of our instructors here at AoPS, Jeremy Copeland.
rrusczyk
2010-02-10 19:04:43
Before joining us at AoPS, he was working at MIT, teaching some pretty smart students there! He has a PhD from the University of Chicago, so he's very happy to now be living in San Diego.
Before joining us at AoPS, he was working at MIT, teaching some pretty smart students there! He has a PhD from the University of Chicago, so he's very happy to now be living in San Diego.
rrusczyk
2010-02-10 19:04:56
He's smarter than all of us put together, but you wouldn't know it if you met him.
He's smarter than all of us put together, but you wouldn't know it if you met him.
rrusczyk
2010-02-10 19:05:15
Assistants can answer questions by whispering to you or by opening a window with you to chat 1-on-1.
Assistants can answer questions by whispering to you or by opening a window with you to chat 1-on-1.
rrusczyk
2010-02-10 19:05:33
Please also remember that the purpose of this Math Jam is to work through the solutions to AMC problems, and not to merely present the answers. "Working through the solutions" includes discussing problem-solving tactics. So please, when a question is posted, do not simply respond with the answer. That's not why we're here. We're going to work through the problems step-by-step, and people who post comments that skip key steps or jump ahead in the problem, without providing explanation or motivation, are going to be ignored.
Please also remember that the purpose of this Math Jam is to work through the solutions to AMC problems, and not to merely present the answers. "Working through the solutions" includes discussing problem-solving tactics. So please, when a question is posted, do not simply respond with the answer. That's not why we're here. We're going to work through the problems step-by-step, and people who post comments that skip key steps or jump ahead in the problem, without providing explanation or motivation, are going to be ignored.
ksun48
2010-02-10 19:05:36
what about mr. sato
what about mr. sato
rrusczyk
2010-02-10 19:06:07
nsato helped prepare the Math Jam, but he has another class to teach this evening. Even Naoki cannot teach two classes at once!
nsato helped prepare the Math Jam, but he has another class to teach this evening. Even Naoki cannot teach two classes at once!
rrusczyk
2010-02-10 19:06:20
The Math Jam will proceed as follows:
The Math Jam will proceed as follows:
rrusczyk
2010-02-10 19:06:34
We will work the last 5 problems from the AMC 10A, then the last 5 problems from the AMC 12A. After that, time and my hands permitting, I will take requests for some other problems for discussion.
We will work the last 5 problems from the AMC 10A, then the last 5 problems from the AMC 12A. After that, time and my hands permitting, I will take requests for some other problems for discussion.
rrusczyk
2010-02-10 19:07:05
We'll get started with....
We'll get started with....
rrusczyk
2010-02-10 19:07:15

ritwik_anand
2010-02-10 19:07:32
vieta's equations
vieta's equations
BOGTRO
2010-02-10 19:07:32
Vieta
Vieta
nikeballa96
2010-02-10 19:07:32
vieta's!
vieta's!
MathTwo
2010-02-10 19:07:32
vieta's formulas
vieta's formulas
Just_Beginner
2010-02-10 19:07:32
Use vieta's formulas
Use vieta's formulas
Lord.of.AMC
2010-02-10 19:07:32
use vieta
use vieta
MrBob
2010-02-10 19:07:32
use vietas formulas
use vietas formulas
rrusczyk
2010-02-10 19:07:40
Let's see how.
Let's see how.
rrusczyk
2010-02-10 19:07:41
Let the zeros be r,s,t. What do we know about r,s,t?
Let the zeros be r,s,t. What do we know about r,s,t?
darkdieuguerre
2010-02-10 19:08:02
r+s+t = a
r+s+t = a
simplyMathLete
2010-02-10 19:08:02
rst = 2010
rst = 2010
BOGTRO
2010-02-10 19:08:02
rst=2010
rst=2010
mz94
2010-02-10 19:08:02
rst=2010
rst=2010
flare
2010-02-10 19:08:02
rst=2010
rst=2010
EricMathPath09
2010-02-10 19:08:02
rst=2010
rst=2010
Visitor03
2010-02-10 19:08:02
they're sum is a
they're sum is a
Just_Beginner
2010-02-10 19:08:02
r+s+t=a
r+s+t=a
PhireKaLk6781
2010-02-10 19:08:02
r+s+t=a
r+s+t=a
fortenforge
2010-02-10 19:08:02
r + s + t = a and rst = 2010
r + s + t = a and rst = 2010
Truffles
2010-02-10 19:08:02
rst = 2010 and r + s + t = a.
rst = 2010 and r + s + t = a.
rrusczyk
2010-02-10 19:08:08
We have rst = 2010 and r+s+t = a.
We have rst = 2010 and r+s+t = a.
rrusczyk
2010-02-10 19:08:18
This comes from some of you are calling "Vieta"
This comes from some of you are calling "Vieta"
rrusczyk
2010-02-10 19:08:23

rrusczyk
2010-02-10 19:08:33
The constant terms give us rst = 2010, and the x^2 terms give us r+s+t = a.
The constant terms give us rst = 2010, and the x^2 terms give us r+s+t = a.
rrusczyk
2010-02-10 19:08:47
This approach to relating roots and coefficients is what we call "Vieta"
This approach to relating roots and coefficients is what we call "Vieta"
ksun48
2010-02-10 19:08:54
we need to minimize r+s+t
we need to minimize r+s+t
rrusczyk
2010-02-10 19:08:58
We know that r,s,t are positive integers, so we must minimize r+s+t such that rst=2010.
We know that r,s,t are positive integers, so we must minimize r+s+t such that rst=2010.
prezcoin
2010-02-10 19:09:08
factorization of 2010= 2*3*5*67
factorization of 2010= 2*3*5*67
nikeballa96
2010-02-10 19:09:08

abcak
2010-02-10 19:09:08
2010=2*3*5*67
2010=2*3*5*67
Just_Beginner
2010-02-10 19:09:08
2010=67*3*2*5
2010=67*3*2*5
PowerOfPi
2010-02-10 19:09:08
prime factorize 2010 to get 2*3*5*67
prime factorize 2010 to get 2*3*5*67
rrusczyk
2010-02-10 19:09:16

rrusczyk
2010-02-10 19:09:23
Now what?
Now what?
Truffles
2010-02-10 19:09:48
2010 = (67)(3)(2)(5). In order to minimize r + s + t we use x,y,z as 67, 6, and 5. 67 + 6 + 5 = 78
2010 = (67)(3)(2)(5). In order to minimize r + s + t we use x,y,z as 67, 6, and 5. 67 + 6 + 5 = 78
sabretooth
2010-02-10 19:09:48
2*3 + 5 + 67
2*3 + 5 + 67
henrypickle
2010-02-10 19:09:48
67 is a zero because 67*another factor would be too large
67 is a zero because 67*another factor would be too large
nikeballa96
2010-02-10 19:09:48
we know r=67, so find s and t such that s+t is minimized.
we know r=67, so find s and t such that s+t is minimized.
EricMathPath09
2010-02-10 19:09:48
2010=2*3*5*67. The smallest possible sum is 6+5+67=11+67=78.
2010=2*3*5*67. The smallest possible sum is 6+5+67=11+67=78.
ksun48
2010-02-10 19:09:48
use 5, 6, and 67
use 5, 6, and 67
flare
2010-02-10 19:09:48
smallest numbers possible are 6, 5, and 67... now sum them for the answer
smallest numbers possible are 6, 5, and 67... now sum them for the answer
megahertz
2010-02-10 19:09:48
to minimize, multiply the two smallest to get 6*5*67
to minimize, multiply the two smallest to get 6*5*67
rrusczyk
2010-02-10 19:09:53
To minimize r+s+t, we need 67 to be one of the roots (since multiplying 67 by anything will way increase r+s+t!) Then, we're left with two roots with product 2*3*5. A little intuition, or some trial and error, will tell us that we want one root to be 2*3 and the other to be 5, giving us a sum of 5+6+67 = 78.
To minimize r+s+t, we need 67 to be one of the roots (since multiplying 67 by anything will way increase r+s+t!) Then, we're left with two roots with product 2*3*5. A little intuition, or some trial and error, will tell us that we want one root to be 2*3 and the other to be 5, giving us a sum of 5+6+67 = 78.
rrusczyk
2010-02-10 19:10:02
This must be the right answer, since there are no smaller answers to check!!
This must be the right answer, since there are no smaller answers to check!!
rrusczyk
2010-02-10 19:10:47
A quick note: we have about 250 people in the room right now. So, those of you checking this out to see what our classes are like: this is not what our classes are like in general -- we won't be able to handle lots of questions tonight, for example.
A quick note: we have about 250 people in the room right now. So, those of you checking this out to see what our classes are like: this is not what our classes are like in general -- we won't be able to handle lots of questions tonight, for example.
MathSuperStar
2010-02-10 19:10:50
so it would be A
so it would be A
rrusczyk
2010-02-10 19:10:53
Exactly.
Exactly.
rrusczyk
2010-02-10 19:11:06
Also, I won't be able to answer general questions about our courses while doing the Math Jam.
Also, I won't be able to answer general questions about our courses while doing the Math Jam.
rrusczyk
2010-02-10 19:11:21
You can email us at classes@artofproblemsolving.com to ask questions about specific courses.
You can email us at classes@artofproblemsolving.com to ask questions about specific courses.
rrusczyk
2010-02-10 19:11:35
You can learn about Vieta in our Intermediate Algebra text or Algebra 3 course.
You can learn about Vieta in our Intermediate Algebra text or Algebra 3 course.
rrusczyk
2010-02-10 19:11:45
Now, on to Problem 22!
Now, on to Problem 22!
rrusczyk
2010-02-10 19:11:48

rrusczyk
2010-02-10 19:12:09
One of my favorite counting strategies is what I call "constructive counting" -- we think about how to construct one instance of what we are trying to count.
One of my favorite counting strategies is what I call "constructive counting" -- we think about how to construct one instance of what we are trying to count.
rrusczyk
2010-02-10 19:12:26
How do we end up with a triangle entirely inside the circle?
How do we end up with a triangle entirely inside the circle?
sabretooth
2010-02-10 19:12:49
You need six points to make a single triangle under those conditions.
You need six points to make a single triangle under those conditions.
MathTwo
2010-02-10 19:12:49
three intersecting chords form a interior triangle
three intersecting chords form a interior triangle
aleph0
2010-02-10 19:12:49
you need 6 points on the outside
you need 6 points on the outside
fortenforge
2010-02-10 19:12:49
We need 3 lines that each intersect each other inside the circle.
We need 3 lines that each intersect each other inside the circle.
straw1boy
2010-02-10 19:12:49
pick three chords
pick three chords
rrusczyk
2010-02-10 19:12:57
Such a triangle is formed by three lines:
Such a triangle is formed by three lines:
rrusczyk
2010-02-10 19:13:02

rrusczyk
2010-02-10 19:13:11
What's a natural question to ask ourselves about the triangle we've created with these three lines connecting these six points on the circle?
What's a natural question to ask ourselves about the triangle we've created with these three lines connecting these six points on the circle?
Just_Beginner
2010-02-10 19:13:34
is it unique?
is it unique?
pirox
2010-02-10 19:13:34
is there another?
is there another?
ksun48
2010-02-10 19:13:34
what if they intersect?
what if they intersect?
aleph0
2010-02-10 19:13:34
does it always exist
does it always exist
agentcx
2010-02-10 19:13:34
when do the three lines intersect?
when do the three lines intersect?
rrusczyk
2010-02-10 19:13:51
We should wonder both of these.
We should wonder both of these.
rrusczyk
2010-02-10 19:14:03
First, is this the only triangle we could make with these 6 points?
First, is this the only triangle we could make with these 6 points?
pirox
2010-02-10 19:14:24
ya
ya
sparkle123
2010-02-10 19:14:24
yes
yes
sabretooth
2010-02-10 19:14:24
yes
yes
challenger
2010-02-10 19:14:24
yes
yes
Wikipedian1337
2010-02-10 19:14:24
yes, each is paired w/ the opposite point
yes, each is paired w/ the opposite point
rrusczyk
2010-02-10 19:14:29
If we connect two adjacent points, neither of the other two lines can intersect the first edge we have drawn:
If we connect two adjacent points, neither of the other two lines can intersect the first edge we have drawn:
rrusczyk
2010-02-10 19:14:36

rrusczyk
2010-02-10 19:14:42
If we connect two points that have only one point between them, then there is only one more line that we can draw that will intersect the first line we have drawn:
If we connect two points that have only one point between them, then there is only one more line that we can draw that will intersect the first line we have drawn:
rrusczyk
2010-02-10 19:14:48

Just_Beginner
2010-02-10 19:15:04
they have to be 3 vertices away
they have to be 3 vertices away
rrusczyk
2010-02-10 19:15:09
Therefore, to get a triangle from connecting 6 points, we must connect each pair of opposite points.
Therefore, to get a triangle from connecting 6 points, we must connect each pair of opposite points.
rrusczyk
2010-02-10 19:15:17
Will we always get a triangle when we take 6 points on the circle, and connect each pair of opposite points?
Will we always get a triangle when we take 6 points on the circle, and connect each pair of opposite points?
snowpaw
2010-02-10 19:15:48
the problems says that no 3 lines are collinear
the problems says that no 3 lines are collinear
darkdieuguerre
2010-02-10 19:15:48
yes because no three lines intersect at a single point
yes because no three lines intersect at a single point
aleph0
2010-02-10 19:15:48
yes because no three lines are concurrent
yes because no three lines are concurrent
MathTwo
2010-02-10 19:15:48
yeah we should
yeah we should
happyof
2010-02-10 19:15:48
yes
yes
Lord.of.AMC
2010-02-10 19:15:48
yes
yes
joshim5
2010-02-10 19:15:48
Yes, we will
Yes, we will
ytrewq
2010-02-10 19:15:48
yes, no three chords intersect at a single point
yes, no three chords intersect at a single point
rrusczyk
2010-02-10 19:15:53
Yes! If we do so, each pair of lines must intersect in the circle. These three intersections then form our triangle.
Yes! If we do so, each pair of lines must intersect in the circle. These three intersections then form our triangle.
abcak
2010-02-10 19:15:59
yes. So that means for each set of 6 points, there is 1 possible triangle
yes. So that means for each set of 6 points, there is 1 possible triangle
rrusczyk
2010-02-10 19:16:03
Exactly!
Exactly!
megahertz
2010-02-10 19:16:16
so we count how many sets of 6 points there are
so we count how many sets of 6 points there are
rrusczyk
2010-02-10 19:16:19
So, for every group of 6 points, we get exactly one triangle. Now our problem is simply counting how many groups of 6 points there are among the initial 8 points.
So, for every group of 6 points, we get exactly one triangle. Now our problem is simply counting how many groups of 6 points there are among the initial 8 points.
Just_Beginner
2010-02-10 19:16:39
how many ways to select 6 points from the 8 given is the new problem
how many ways to select 6 points from the 8 given is the new problem
nearsymmetric
2010-02-10 19:16:39
so 8 choose 6
so 8 choose 6
fortenforge
2010-02-10 19:16:39
So we need to find the number of 6 points to be chosen from 8
So we need to find the number of 6 points to be chosen from 8
MathTwo
2010-02-10 19:16:39
then 28 (A) is our answer
then 28 (A) is our answer
aleph0
2010-02-10 19:16:39
so we need 8 choose 6
so we need 8 choose 6
simplyMathLete
2010-02-10 19:16:39
okay, so now 8C6?
okay, so now 8C6?
mathrat
2010-02-10 19:16:39
8 choose 6?
8 choose 6?
megahertz
2010-02-10 19:16:39
so 8C6?
so 8C6?
alex960531
2010-02-10 19:16:39
8C6 = 28 (A)
8C6 = 28 (A)
baldcypress
2010-02-10 19:16:39
8C2=28 A
8C2=28 A
challenger
2010-02-10 19:16:39
28
28
pumpkinpi
2010-02-10 19:16:41
So we have 8C6 = 8C2 = 56/2 = 28
So we have 8C6 = 8C2 = 56/2 = 28
rrusczyk
2010-02-10 19:16:46
This is simply C(8,6) = C(8,2) = 8*7/2 = 28, so there are 28 triangles.
This is simply C(8,6) = C(8,2) = 8*7/2 = 28, so there are 28 triangles.
LincolnMS nerd herd
2010-02-10 19:17:26
and so the answer is A and problem solved =)
and so the answer is A and problem solved =)
rrusczyk
2010-02-10 19:17:29
Exactly.
Exactly.
ss5188
2010-02-10 19:17:37
but what about the triangles that have their vertices on the chords' ends?
but what about the triangles that have their vertices on the chords' ends?
rrusczyk
2010-02-10 19:17:45
The problem tells us not to count these.
The problem tells us not to count these.
megahertz
2010-02-10 19:18:06
what if the problem did?
what if the problem did?
rrusczyk
2010-02-10 19:18:22
Interesting question -- I challenge you all to think about that on your own :)
Interesting question -- I challenge you all to think about that on your own :)
rrusczyk
2010-02-10 19:18:38
All right, let's take a look at the next problem
All right, let's take a look at the next problem
rrusczyk
2010-02-10 19:18:40

rrusczyk
2010-02-10 19:19:06
How can we get a feel for the problem?
How can we get a feel for the problem?
AdithyaGanesh
2010-02-10 19:19:20
If you play around with it a bit, you'll notice that if you multiply the fractions through, many numbers get cancelled out.
If you play around with it a bit, you'll notice that if you multiply the fractions through, many numbers get cancelled out.
megahertz
2010-02-10 19:19:20
try a small case of (n)
try a small case of (n)
PowerOfPi
2010-02-10 19:19:20
Try small cases.
Try small cases.
limac
2010-02-10 19:19:20
simple cases
simple cases
sabretooth
2010-02-10 19:19:23
Try smaller examples first.
Try smaller examples first.
KingJames
2010-02-10 19:19:24
Try small values of n
Try small values of n
rrusczyk
2010-02-10 19:19:33
We try small examples, since this will help us:
We try small examples, since this will help us:
mathcountsloser
2010-02-10 19:19:36
Understand it??
Understand it??
LincolnMS nerd herd
2010-02-10 19:19:38
i would start small and work my way up
i would start small and work my way up
rrusczyk
2010-02-10 19:19:41
What is P(1)?
What is P(1)?
nsreenivas
2010-02-10 19:19:51
1/2
1/2
challenger
2010-02-10 19:19:51
1/2
1/2
PowerOfPi
2010-02-10 19:19:51
1/2
1/2
pirox
2010-02-10 19:19:51
1/2
1/2
ksun48
2010-02-10 19:19:51
1/2
1/2
darkdieuguerre
2010-02-10 19:19:51
1/2
1/2
Lord.of.AMC
2010-02-10 19:19:51
1/2
1/2
vcez
2010-02-10 19:19:51
1/2
1/2
xoangieexo
2010-02-10 19:19:51
1/2
1/2
xxrxxhxx
2010-02-10 19:19:51
1/2
1/2
Lord.of.AMC
2010-02-10 19:19:51
1/2
1/2
agentcx
2010-02-10 19:19:51
1/2
1/2
mathluver2.718
2010-02-10 19:19:51
1/2
1/2
rrusczyk
2010-02-10 19:19:57
We have P(1) = 1/2, since the first box has 1 red, 1 white.
We have P(1) = 1/2, since the first box has 1 red, 1 white.
rrusczyk
2010-02-10 19:20:01
What about P(2)?
What about P(2)?
Visitor03
2010-02-10 19:20:29
1/6
1/6
aerrowfinn72
2010-02-10 19:20:29
1/6
1/6
fortenforge
2010-02-10 19:20:29
1/2 * 1/3
1/2 * 1/3
megahertz
2010-02-10 19:20:29
1/2*1/3=1/6
1/2*1/3=1/6
mz94
2010-02-10 19:20:29
1/2*1/3=1/6
1/2*1/3=1/6
AdithyaGanesh
2010-02-10 19:20:29
1/6, or 1/2*1/3.
1/6, or 1/2*1/3.
LincolnMS nerd herd
2010-02-10 19:20:29
1/2*1/3=1/6
1/2*1/3=1/6
simplyMathLete
2010-02-10 19:20:31
the second would be 1/2*1/3 = 1/6
the second would be 1/2*1/3 = 1/6
rrusczyk
2010-02-10 19:20:34
We have P(2) = 1/2 * 1/3, since she must choose white, then red.
We have P(2) = 1/2 * 1/3, since she must choose white, then red.
rrusczyk
2010-02-10 19:20:36
P(3)?
P(3)?
henrypickle
2010-02-10 19:20:59
1/2*2/3*1/4
1/2*2/3*1/4
Just_Beginner
2010-02-10 19:21:00
1/2*2/3*1/4
1/2*2/3*1/4
mgao
2010-02-10 19:21:00
1/2*2/3*1/4=1/12
1/2*2/3*1/4=1/12
ytrewq
2010-02-10 19:21:00
1/2*2/3*1/4=1/12
1/2*2/3*1/4=1/12
jxl28
2010-02-10 19:21:00
(1/2)(2/3)(1/4)=1/12
(1/2)(2/3)(1/4)=1/12
Visitor03
2010-02-10 19:21:00
1/2 * 2/3 * 1/4
1/2 * 2/3 * 1/4
rrusczyk
2010-02-10 19:21:03
We get P(3) = 1/2 * 2/3 * 1/4 = 1/3 * 1/4, since she must choose white-white-red.
We get P(3) = 1/2 * 2/3 * 1/4 = 1/3 * 1/4, since she must choose white-white-red.
rrusczyk
2010-02-10 19:21:10
P(4)?
P(4)?
challenger
2010-02-10 19:21:29
1/20
1/20
ksun48
2010-02-10 19:21:29
1/20
1/20
Just_Beginner
2010-02-10 19:21:29
1/2*2/3*3/4*1/5
1/2*2/3*3/4*1/5
lolFTW
2010-02-10 19:21:29
1/4*1/5=1/20
1/4*1/5=1/20
fortenforge
2010-02-10 19:21:29
1/2 * 2/3 * 3/4 * 1/5
1/2 * 2/3 * 3/4 * 1/5
The Archer
2010-02-10 19:21:29
(1/2)(2/3)(3/4)(1/5)=1/20
(1/2)(2/3)(3/4)(1/5)=1/20
jxl28
2010-02-10 19:21:29
(1/2)(2/3)(3/4)(1/5)=1/20
(1/2)(2/3)(3/4)(1/5)=1/20
wiiwilwin
2010-02-10 19:21:32
(1/2)(2/3)(3/4)(1/5)
(1/2)(2/3)(3/4)(1/5)
rrusczyk
2010-02-10 19:21:36
We have P(4) = 1/2 * 2/3 * 3/4 * 1/5 = 1/4 * 1/5.
We have P(4) = 1/2 * 2/3 * 3/4 * 1/5 = 1/4 * 1/5.
rrusczyk
2010-02-10 19:21:45
Aha! What do we guess P(n) will be?
Aha! What do we guess P(n) will be?
megahertz
2010-02-10 19:22:15
numerators and denominators cancel out
numerators and denominators cancel out
prezcoin
2010-02-10 19:22:15

oxfz16
2010-02-10 19:22:15
the pattern is 1/[n(n+1)]
the pattern is 1/[n(n+1)]
AdithyaGanesh
2010-02-10 19:22:15
So now, P(n) = 1/[n(n+1)].
So now, P(n) = 1/[n(n+1)].
PowerOfPi
2010-02-10 19:22:15
1/(n(n+1))
1/(n(n+1))
dolk
2010-02-10 19:22:15
1/(n(n+1))
1/(n(n+1))
Lord.of.AMC
2010-02-10 19:22:15
the product P(n) telescopes to 1/n(n + 1)
the product P(n) telescopes to 1/n(n + 1)
mathluver2.718
2010-02-10 19:22:15
1/(n)(n+1)
1/(n)(n+1)
7thsamurai
2010-02-10 19:22:15
1/(n * (n+1))
1/(n * (n+1))
rrusczyk
2010-02-10 19:22:25

mgao
2010-02-10 19:22:47
so we need n(n+1)>2010
so we need n(n+1)>2010
nikeballa96
2010-02-10 19:22:47
we need to find the smallest n such that n(n+1)>2010
we need to find the smallest n such that n(n+1)>2010
flare
2010-02-10 19:23:00
so we have to find the smallest value such taht n^2+n is greater than 2010
so we have to find the smallest value such taht n^2+n is greater than 2010
ksun48
2010-02-10 19:23:03
now find 1/(n*(n+1)<1/2010
now find 1/(n*(n+1)<1/2010
rrusczyk
2010-02-10 19:23:06
And that would be?
And that would be?
vcez
2010-02-10 19:23:27
n=45
n=45
aerrowfinn72
2010-02-10 19:23:27
45
45
fortenforge
2010-02-10 19:23:27
45
45
RobRoobiks
2010-02-10 19:23:27
45
45
henrypickle
2010-02-10 19:23:27
45
45
abcak
2010-02-10 19:23:27
we can easily see that is 45
we can easily see that is 45
steve123456
2010-02-10 19:23:27
45
45
gomath888
2010-02-10 19:23:27
45
45
simplyMathLete
2010-02-10 19:23:27
45*46 > 2010
45*46 > 2010
msinghal
2010-02-10 19:23:27
45
45
rrusczyk
2010-02-10 19:23:35
So, we just need n^2 + n > 2010, and we'll get P(n) < 1/2010. Obviously 50^2 + 50 is way more than 2010, so (A) must be the answer. We can do a quick check and note that 44^2 + 44 = 1980, and 45^2 + 45 = 2070, so 45 is indeed correct.
So, we just need n^2 + n > 2010, and we'll get P(n) < 1/2010. Obviously 50^2 + 50 is way more than 2010, so (A) must be the answer. We can do a quick check and note that 44^2 + 44 = 1980, and 45^2 + 45 = 2070, so 45 is indeed correct.
challenger
2010-02-10 19:23:54
A
A
ksun48
2010-02-10 19:23:54
A again?
A again?
MathSuperStar
2010-02-10 19:23:54
its A
its A
flare
2010-02-10 19:23:54
A is, once again, the answer
A is, once again, the answer
pirox
2010-02-10 19:23:54
so it's (A) 45 =P
so it's (A) 45 =P
rrusczyk
2010-02-10 19:23:57
The answer again is A.
The answer again is A.
rrusczyk
2010-02-10 19:24:22
(And again a reminder -- we won't get to every one of your questions tonight like we do in our regular classes. There are now nearly 300 of you here!)
(And again a reminder -- we won't get to every one of your questions tonight like we do in our regular classes. There are now nearly 300 of you here!)
rrusczyk
2010-02-10 19:24:29
Time for number 24
Time for number 24
rrusczyk
2010-02-10 19:24:39

mathcountsloser
2010-02-10 19:24:46
#24 is very advanced for the amc 10
#24 is very advanced for the amc 10
ksun48
2010-02-10 19:24:54
the hardest problem on the 10
the hardest problem on the 10
rrusczyk
2010-02-10 19:25:02
I agree. This is the hardest problem on the test.
I agree. This is the hardest problem on the test.
rrusczyk
2010-02-10 19:25:06
OK, before we get into the details of a solution, suppose you were given 6 minutes to get an answer. What would you do?
OK, before we get into the details of a solution, suppose you were given 6 minutes to get an answer. What would you do?
TheWorstPlayer
2010-02-10 19:25:34
bash?
bash?
rrusczyk
2010-02-10 19:25:43
That's what I'd try if I hadn't done this:
That's what I'd try if I hadn't done this:
RobRoobiks
2010-02-10 19:25:47
skip it
skip it
aleph0
2010-02-10 19:25:47
break my pencil
break my pencil
rrusczyk
2010-02-10 19:25:56
Just start multiplying away, you only want the last two digits, and any time you hit a factor of 5, you pull a factor of 2 out of some other number. For example, when working through the 20s, you'd multiply by 23, then 6, then 26, where you pair two of the 2's from 24 with the 5's in 25, and just throw them away.
Just start multiplying away, you only want the last two digits, and any time you hit a factor of 5, you pull a factor of 2 out of some other number. For example, when working through the 20s, you'd multiply by 23, then 6, then 26, where you pair two of the 2's from 24 with the 5's in 25, and just throw them away.
rrusczyk
2010-02-10 19:26:02
This isn't a terribly revealing way to do the problem, but it will work, and for most people (including me), it will be way faster than finding the "smart" way to do the problem.
This isn't a terribly revealing way to do the problem, but it will work, and for most people (including me), it will be way faster than finding the "smart" way to do the problem.
rrusczyk
2010-02-10 19:26:14
OK. Moving on to trying to find the smart way to do it...
OK. Moving on to trying to find the smart way to do it...
rrusczyk
2010-02-10 19:26:24
After we lop off all the trailing zeroes, we want the last two digits that remain. Let's let n be the number that's left after clipping off all the trailing zeroes. We want the remainder when n is divided by 100. We sometimes refer to this as "n mod 100".
After we lop off all the trailing zeroes, we want the last two digits that remain. Let's let n be the number that's left after clipping off all the trailing zeroes. We want the remainder when n is divided by 100. We sometimes refer to this as "n mod 100".
rrusczyk
2010-02-10 19:26:33
(If you aren't familiar with mods, check out our Introduction to Number Theory book or class; we unfortunately don't have time tonight to teach all the fundamentals of mods. The rest of this solution will be pretty impossible to follow if you aren't proficient with mods.)
(If you aren't familiar with mods, check out our Introduction to Number Theory book or class; we unfortunately don't have time tonight to teach all the fundamentals of mods. The rest of this solution will be pretty impossible to follow if you aren't proficient with mods.)
rrusczyk
2010-02-10 19:26:42
Now, might we attack something simpler than mod 100 for starters?
Now, might we attack something simpler than mod 100 for starters?
mathcountsloser
2010-02-10 19:26:53
I used mod 25
I used mod 25
mgao
2010-02-10 19:26:54
mod 25
mod 25
PhireKaLk6781
2010-02-10 19:26:54
mod 25
mod 25
vjnmath
2010-02-10 19:26:54
mod 25
mod 25
rrusczyk
2010-02-10 19:27:03
We might start with mod 25; since 100 = 25 * 4, and if we can find the remainder upon division by 25, we've at least narrowed down our choices considerably.
We might start with mod 25; since 100 = 25 * 4, and if we can find the remainder upon division by 25, we've at least narrowed down our choices considerably.
rrusczyk
2010-02-10 19:27:08
Why else does 25 seem like a convenient choice?
Why else does 25 seem like a convenient choice?
rrusczyk
2010-02-10 19:27:34
Why is convenient for this particular problem?
Why is convenient for this particular problem?
rrusczyk
2010-02-10 19:28:33
If we start cranking out the factorials, what do we see that's convenient?
If we start cranking out the factorials, what do we see that's convenient?
aleph0
2010-02-10 19:28:55
a lot of stuff cancels
a lot of stuff cancels
rrusczyk
2010-02-10 19:28:58
Why?
Why?
rrusczyk
2010-02-10 19:29:21
What I'm driving at is 4!
What I'm driving at is 4!
rrusczyk
2010-02-10 19:29:27
What is 4! in mod 25?
What is 4! in mod 25?
darkdieuguerre
2010-02-10 19:29:47
-1
-1
prezcoin
2010-02-10 19:29:47
-1
-1
simplyMathLete
2010-02-10 19:29:47
-1
-1
EricMathPath09
2010-02-10 19:29:47
-1
-1
abcak
2010-02-10 19:29:47
-1...holy cow
-1...holy cow
mathcountsloser
2010-02-10 19:29:47
or -1.
or -1.
PhireKaLk6781
2010-02-10 19:29:47
or it can be -1 mod 25
or it can be -1 mod 25
rrusczyk
2010-02-10 19:29:49

rrusczyk
2010-02-10 19:29:58
(Again, if you aren't familiar with mods, check out our Introduction to Number Theory book or class; we unfortunately don't have time tonight to teach all the fundamentals of mods. The rest of this solution will be pretty impossible to follow if you aren't proficient with mods.)
(Again, if you aren't familiar with mods, check out our Introduction to Number Theory book or class; we unfortunately don't have time tonight to teach all the fundamentals of mods. The rest of this solution will be pretty impossible to follow if you aren't proficient with mods.)
rrusczyk
2010-02-10 19:30:02
That's pretty nice, since multiplying by -1 is way easy.
That's pretty nice, since multiplying by -1 is way easy.
rrusczyk
2010-02-10 19:30:09
Are there any other groups that have convenient products like this?
Are there any other groups that have convenient products like this?
abcak
2010-02-10 19:30:40
6*7*8*9!
6*7*8*9!
xxrxxhxx
2010-02-10 19:30:40
So then we have grouping, 1*2*3*4, 6*7*8*9, 11*12*13*14... and on and on
So then we have grouping, 1*2*3*4, 6*7*8*9, 11*12*13*14... and on and on
darkdieuguerre
2010-02-10 19:30:40
(5k+1)(5k+2)(5k+3)(5k+4) in general
(5k+1)(5k+2)(5k+3)(5k+4) in general
ksun48
2010-02-10 19:30:40
6*7*8*9, 11*12*13*14
6*7*8*9, 11*12*13*14
PowerOfPi
2010-02-10 19:30:42
9*8*7*6
9*8*7*6
rrusczyk
2010-02-10 19:30:48

rrusczyk
2010-02-10 19:30:55
That's really nice. Similarly, we can group all the numbers that aren't divisible by 5 into groups of 4 whose products are congruent to -1 mod 25. So, we have
That's really nice. Similarly, we can group all the numbers that aren't divisible by 5 into groups of 4 whose products are congruent to -1 mod 25. So, we have
rrusczyk
2010-02-10 19:31:00

rrusczyk
2010-02-10 19:31:06

nmani2
2010-02-10 19:31:11
you leave out all the 5s!
you leave out all the 5s!
ksun48
2010-02-10 19:31:31
then divide by 2's for the 5's
then divide by 2's for the 5's
gopack25
2010-02-10 19:31:31
u can leave out all the 5
u can leave out all the 5
Jesse
2010-02-10 19:31:31
but they cancel with the 2s!
but they cancel with the 2s!
rrusczyk
2010-02-10 19:31:39
Of course, we're going to have to give up some of the factors of 2 in this product to combine with factors of 5 in the other numbers to make the 10s that give us the 0s we'll lop off at the end. How many 2's are we going to use doing this?
Of course, we're going to have to give up some of the factors of 2 in this product to combine with factors of 5 in the other numbers to make the 10s that give us the 0s we'll lop off at the end. How many 2's are we going to use doing this?
aleph0
2010-02-10 19:31:54
21
21
nikeballa96
2010-02-10 19:31:54
21.
21.
nikeballa96
2010-02-10 19:31:54
21.
21.
33286
2010-02-10 19:31:54
21
21
challenger
2010-02-10 19:31:54
21
21
lxu1
2010-02-10 19:31:58
21
21
rrusczyk
2010-02-10 19:32:02
We have to count the factors of 5 in 5*10*15*...*90.
We have to count the factors of 5 in 5*10*15*...*90.
rrusczyk
2010-02-10 19:32:07
There are 3 multiples of 25, and 18 total multiples of 5, for a total of 18+3 = 21 factors of 5.
There are 3 multiples of 25, and 18 total multiples of 5, for a total of 18+3 = 21 factors of 5.
rrusczyk
2010-02-10 19:32:13
So, we have P = 2^{21} * Q, where Q is the number we really care about, since we'll use up the 21 2s making 10s. (We'll get some 2's back when we multiply out the multiples of 5 -- we'll get to that shortly.)
So, we have P = 2^{21} * Q, where Q is the number we really care about, since we'll use up the 21 2s making 10s. (We'll get some 2's back when we multiply out the multiples of 5 -- we'll get to that shortly.)
rrusczyk
2010-02-10 19:32:35

aleph0
2010-02-10 19:33:16
and nicely 2^21=2 mod 25
and nicely 2^21=2 mod 25
happyof
2010-02-10 19:33:16
2^10=-1 mod 25
2^10=-1 mod 25
rrusczyk
2010-02-10 19:33:34

rrusczyk
2010-02-10 19:33:49
So, what is Q(mod 25)?
So, what is Q(mod 25)?
gomath888
2010-02-10 19:34:00
q=13
q=13
SuitcaseAsian
2010-02-10 19:34:00
13?
13?
aleph0
2010-02-10 19:34:00
13
13
prezcoin
2010-02-10 19:34:00
13
13
simplyMathLete
2010-02-10 19:34:05
so Q = 13(mod25)
so Q = 13(mod25)
rrusczyk
2010-02-10 19:34:10

rrusczyk
2010-02-10 19:34:19
We've now learned that the product of all the numbers from 1 to 90 without factors of 5 is 2^{21} times Q, where Q is congruent to 13 mod 25.
We've now learned that the product of all the numbers from 1 to 90 without factors of 5 is 2^{21} times Q, where Q is congruent to 13 mod 25.
rrusczyk
2010-02-10 19:34:36
(If you're totally lost on this problem, don't feel bad -- it's by far the hardest problem on this test, I think)
(If you're totally lost on this problem, don't feel bad -- it's by far the hardest problem on this test, I think)
rrusczyk
2010-02-10 19:34:45
Now, we have to deal with 5*10*15*...*90. Of course, we have to strip out the factors of 5. We know there are 21 of these, and we'll pair them with the 21 2's we took out of P.
Now, we have to deal with 5*10*15*...*90. Of course, we have to strip out the factors of 5. We know there are 21 of these, and we'll pair them with the 21 2's we took out of P.
rrusczyk
2010-02-10 19:35:02

rrusczyk
2010-02-10 19:35:12
So, we want to know what
So, we want to know what
rrusczyk
2010-02-10 19:35:19

rrusczyk
2010-02-10 19:35:22
is mod 25
is mod 25
33286
2010-02-10 19:35:36
we have more -1s mod 25
we have more -1s mod 25
agentcx
2010-02-10 19:35:36
doesnt that go back to the congruencies we discussed earlier?
doesnt that go back to the congruencies we discussed earlier?
rrusczyk
2010-02-10 19:35:56
Yep, we have already dealt with a lot of these already.
Yep, we have already dealt with a lot of these already.
simplyMathLete
2010-02-10 19:36:01
24 = -1(mod25)
24 = -1(mod25)
nmani2
2010-02-10 19:36:03
1*2*3*4 = -1 and doesn't this become a whole lot of -1s
1*2*3*4 = -1 and doesn't this become a whole lot of -1s
2009
2010-02-10 19:36:16
many things cancel out!
many things cancel out!
rrusczyk
2010-02-10 19:36:21
Fortunately, we've already dealt with the first three long products, so we can simplify this as
Fortunately, we've already dealt with the first three long products, so we can simplify this as
aleph0
2010-02-10 19:36:25
so we can cancel out 1,2,3,4, 6,7,8,9, and 11,12,13,14 to make -1
so we can cancel out 1,2,3,4, 6,7,8,9, and 11,12,13,14 to make -1
rrusczyk
2010-02-10 19:36:30

megahertz
2010-02-10 19:36:48
how do we get rid of 16*17*18?
how do we get rid of 16*17*18?
rrusczyk
2010-02-10 19:36:54
Just chug it out like we did above.
Just chug it out like we did above.
rrusczyk
2010-02-10 19:37:14
And what do we do with this?
And what do we do with this?
33286
2010-02-10 19:37:36
multiply it by 13
multiply it by 13
agentcx
2010-02-10 19:37:36
combine with the 13 (mod 25) to get 12 (mod 25) (A)
combine with the 13 (mod 25) to get 12 (mod 25) (A)
challenger
2010-02-10 19:37:36
multiple with Q
multiple with Q
nikeballa96
2010-02-10 19:37:45
combine with 13.
combine with 13.
rrusczyk
2010-02-10 19:37:50
So, we multiply this by Q and the resulting product is -13 mod 25, which is equivalent to 12 mod 25. From the 5 choices, it's obvious that the only one that fits is (A), but how else could we eliminate the other 2-digit numbers that are congruent to 12 mod 25?
So, we multiply this by Q and the resulting product is -13 mod 25, which is equivalent to 12 mod 25. From the 5 choices, it's obvious that the only one that fits is (A), but how else could we eliminate the other 2-digit numbers that are congruent to 12 mod 25?
aleph0
2010-02-10 19:38:21
must be divisible by 4
must be divisible by 4
gomath888
2010-02-10 19:38:21
0 mod 4
0 mod 4
rrusczyk
2010-02-10 19:38:26
We know that the last two digits form a multiple of 4, since there are many more factors of 2 in 90! than factors of 5. The only 2-digit multiple of 4 that is congruent to 12 mod 25 is 12 itself, so the answer is 12.
We know that the last two digits form a multiple of 4, since there are many more factors of 2 in 90! than factors of 5. The only 2-digit multiple of 4 that is congruent to 12 mod 25 is 12 itself, so the answer is 12.
rrusczyk
2010-02-10 19:38:35
And, yes, it would almost certainly have been faster to gut it out than to find this solution.
And, yes, it would almost certainly have been faster to gut it out than to find this solution.
rrusczyk
2010-02-10 19:38:53
This solution is pretty tough to find, and very tough to get through if you're not well-versed with mods.
This solution is pretty tough to find, and very tough to get through if you're not well-versed with mods.
sabretooth
2010-02-10 19:38:59
four A's!
four A's!
flare
2010-02-10 19:38:59
A for 4 problems in a row??
A for 4 problems in a row??
rrusczyk
2010-02-10 19:39:02
Yep
Yep
PowerOfPi
2010-02-10 19:39:05
And is that the easiest way?
And is that the easiest way?
aopsaccount2131
2010-02-10 19:39:05
this was the "slick" solution?
this was the "slick" solution?
rrusczyk
2010-02-10 19:39:20
As far as I know; the other solutions I've seen are essentially this.
As far as I know; the other solutions I've seen are essentially this.
bunnygirl
2010-02-10 19:39:53
is this really feasible in the time given?
is this really feasible in the time given?
rrusczyk
2010-02-10 19:40:02
It certainly will cut down on perfect scores...
It certainly will cut down on perfect scores...
rrusczyk
2010-02-10 19:40:07
Let's go to #25
Let's go to #25
rrusczyk
2010-02-10 19:40:12

rrusczyk
2010-02-10 19:40:58
Before I get started -- if you are trying to attend a class tonight besides this math jam, click Classroom again on the sidebar.
Before I get started -- if you are trying to attend a class tonight besides this math jam, click Classroom again on the sidebar.
rrusczyk
2010-02-10 19:41:09
Um, this problem is confusing. How can we make it easier?
Um, this problem is confusing. How can we make it easier?
SuitcaseAsian
2010-02-10 19:41:29
Work backwords from 0
Work backwords from 0
bunnygirl
2010-02-10 19:41:29
work backwards!
work backwards!
PowerOfPi
2010-02-10 19:41:29
Try some simple cases.
Try some simple cases.
PhireKaLk6781
2010-02-10 19:41:29
work backwards from 0
work backwards from 0
GoldenStar
2010-02-10 19:41:29
try an example?
try an example?
nikeballa96
2010-02-10 19:41:29
instead of 8, try 1. and 2. and 3.
instead of 8, try 1. and 2. and 3.
rrusczyk
2010-02-10 19:41:33
Let's try a smaller sequence length. The smallest number that starts a 1-term sequence is easy -- it's 0. 2-term sequences aren't much harder 1,0. Tada.
Let's try a smaller sequence length. The smallest number that starts a 1-term sequence is easy -- it's 0. 2-term sequences aren't much harder 1,0. Tada.
rrusczyk
2010-02-10 19:41:39
How about 3-term sequences? What's the smallest possible starting number?
How about 3-term sequences? What's the smallest possible starting number?
pumpkinpi
2010-02-10 19:41:56
2
2
billy314
2010-02-10 19:41:56
2
2
natiator
2010-02-10 19:41:56
2
2
prezcoin
2010-02-10 19:41:56
2
2
aleph0
2010-02-10 19:41:56
2, 1, 0
2, 1, 0
ksun48
2010-02-10 19:41:56
0, 1, 2
0, 1, 2
sjiang
2010-02-10 19:41:56
Two
Two
rrusczyk
2010-02-10 19:42:00
Oh, yeah, 2 works: 2,1,0.
Oh, yeah, 2 works: 2,1,0.
rrusczyk
2010-02-10 19:42:04
4-term?
4-term?
ritwik_anand
2010-02-10 19:42:21
3,2,1,0
3,2,1,0
nikeballa96
2010-02-10 19:42:21
4 term...3.
4 term...3.
natiator
2010-02-10 19:42:21
3
3
Iggy Iguana
2010-02-10 19:42:21
3
3
nikeballa96
2010-02-10 19:42:21
3.
3.
aleph0
2010-02-10 19:42:21
3,2,1,0
3,2,1,0
ksun48
2010-02-10 19:42:21
3, 2, 1, 0
3, 2, 1, 0
jxl28
2010-02-10 19:42:21
3,2,1,0
3,2,1,0
happyof
2010-02-10 19:42:21
3,2,1,0
3,2,1,0
aerrowfinn72
2010-02-10 19:42:21
3,2,1,0
3,2,1,0
PhireKaLk6781
2010-02-10 19:42:24
nice countdown here...
nice countdown here...
rrusczyk
2010-02-10 19:42:27
4-term?
4-term?
rrusczyk
2010-02-10 19:42:34
3,2,1,0.
3,2,1,0.
rrusczyk
2010-02-10 19:42:37
5-term?
5-term?
challenger
2010-02-10 19:42:55
7,3,2,1,0
7,3,2,1,0
c32r17qp181cd912
2010-02-10 19:42:55
then 7.
then 7.
nikeballa96
2010-02-10 19:42:55
7, 3, 2, 1, 0
7, 3, 2, 1, 0
ksun48
2010-02-10 19:42:55
7, 3, 2, 1, 0
7, 3, 2, 1, 0
PhireKaLk6781
2010-02-10 19:42:55
7,3,2,1,0
7,3,2,1,0
TheWorstPlayer
2010-02-10 19:42:55
7
7
henrypickle
2010-02-10 19:42:55
7
7
darkdieuguerre
2010-02-10 19:42:55
7,3,2,1,0
7,3,2,1,0
EricMathPath09
2010-02-10 19:42:55
7,3,2,1,0
7,3,2,1,0
aleph0
2010-02-10 19:42:55
7,3,2,1,0
7,3,2,1,0
MrBob
2010-02-10 19:42:55
7,3,2,1,0
7,3,2,1,0
rrusczyk
2010-02-10 19:43:03
We have to go all the way up to 7: 7,3,2,1,0.
We have to go all the way up to 7: 7,3,2,1,0.
rrusczyk
2010-02-10 19:43:09
If we start with 4, 5, or 6, the next step is lower than 3 (subtract 4), and we'll get to 0 in less than 5 terms.
If we start with 4, 5, or 6, the next step is lower than 3 (subtract 4), and we'll get to 0 in less than 5 terms.
rrusczyk
2010-02-10 19:43:11
6-term?
6-term?
nikeballa96
2010-02-10 19:43:21
23, 7, 3, 2, 1, 0
23, 7, 3, 2, 1, 0
theprodigy
2010-02-10 19:43:21
23,7,3,2,1,0
23,7,3,2,1,0
nikeballa96
2010-02-10 19:43:21
23, 7, 3, 2, 1, 0
23, 7, 3, 2, 1, 0
nikeballa96
2010-02-10 19:43:21
23, 7, 3, 2, 1, 0
23, 7, 3, 2, 1, 0
rrusczyk
2010-02-10 19:43:35
Yikes! How did we figure out that the next one is 23?
Yikes! How did we figure out that the next one is 23?
rrusczyk
2010-02-10 19:43:49
We know that the second term in the sequence is at least 7. Suppose it is 7. Then, let the first term be k. What could k be?
We know that the second term in the sequence is at least 7. Suppose it is 7. Then, let the first term be k. What could k be?
darkdieuguerre
2010-02-10 19:44:17
we add a minimal square
we add a minimal square
nsreenivas
2010-02-10 19:44:17
9+7=16 which is 4^2 so 7+16=23
9+7=16 which is 4^2 so 7+16=23
EricMathPath09
2010-02-10 19:44:17
+9 gives 16, which doesn't work, so we have to do + 16 to get 23.
+9 gives 16, which doesn't work, so we have to do + 16 to get 23.
nikeballa96
2010-02-10 19:44:17
7+1 doesnt work, nor does 7+4, nor does 7+9. 7+16 does! =]
7+1 doesnt work, nor does 7+4, nor does 7+9. 7+16 does! =]
ss5188
2010-02-10 19:44:17
7+16=23 and 16 is the smallest square to add in the sequence
7+16=23 and 16 is the smallest square to add in the sequence
rrusczyk
2010-02-10 19:44:28
We get to 7 by subtracting a square from k. Moreover, the square we subtract from k is the largest square less than k. So, since k > 7, we already know that the square we subtract cannot be 1 or 4.
We get to 7 by subtracting a square from k. Moreover, the square we subtract from k is the largest square less than k. So, since k > 7, we already know that the square we subtract cannot be 1 or 4.
aerrowfinn72
2010-02-10 19:44:33
cuz adding 9 dusnt work..u get 16 which reduces to 0..so u go to the nxt square
cuz adding 9 dusnt work..u get 16 which reduces to 0..so u go to the nxt square
rrusczyk
2010-02-10 19:44:48
It can't be 9. If we subtracted 9, then the previous number is 7+9 = 16. But we can subtract 16 from that! So, we couldn't have subtracted 9.
It can't be 9. If we subtracted 9, then the previous number is 7+9 = 16. But we can subtract 16 from that! So, we couldn't have subtracted 9.
rrusczyk
2010-02-10 19:44:59
But we could have subtracted 16!
But we could have subtracted 16!
c32r17qp181cd912
2010-02-10 19:45:06
Hence, 7+16=23.
Hence, 7+16=23.
AAFlyer
2010-02-10 19:45:12
so it must have beel 16
so it must have beel 16
rrusczyk
2010-02-10 19:45:20
That gives us:
That gives us:
rrusczyk
2010-02-10 19:45:25
23,7,3,2,1,0.
23,7,3,2,1,0.
rrusczyk
2010-02-10 19:45:49
OK, time for 7-term sequences.
OK, time for 7-term sequences.
rrusczyk
2010-02-10 19:45:57
x,23,7,3,2,1,0
x,23,7,3,2,1,0
rrusczyk
2010-02-10 19:46:05
I see a lot of you jumping to the answer.
I see a lot of you jumping to the answer.
rrusczyk
2010-02-10 19:46:10
Some of you are even right:)
Some of you are even right:)
rrusczyk
2010-02-10 19:46:29
How can we begin to think about this?
How can we begin to think about this?
pumpkinpi
2010-02-10 19:46:49
x - n^2 = 23 for some integer n
x - n^2 = 23 for some integer n
PowerOfPi
2010-02-10 19:46:49
x-a^2=23
x-a^2=23
ksun48
2010-02-10 19:46:49
x-y^2=23
x-y^2=23
rrusczyk
2010-02-10 19:46:55
We want x - n^2 = 23, but we also want n^2 to be the largest square less than x. How can we find that square quickly?
We want x - n^2 = 23, but we also want n^2 to be the largest square less than x. How can we find that square quickly?
rrusczyk
2010-02-10 19:47:03
What else do we know about x?
What else do we know about x?
aopsaccount2131
2010-02-10 19:47:33
less than (n+1) ^ 2
less than (n+1) ^ 2
ytrewq
2010-02-10 19:47:33
(n+1)^2>x
(n+1)^2>x
LincolnMS nerd herd
2010-02-10 19:47:33
it is between n^2 and the next big square number
it is between n^2 and the next big square number
33286
2010-02-10 19:47:33
less than (n+1)^2
less than (n+1)^2
rrusczyk
2010-02-10 19:47:47
We must have x < (n+1)^2.
We must have x < (n+1)^2.
rrusczyk
2010-02-10 19:47:52
How does that help?
How does that help?
mgao
2010-02-10 19:48:28
2n+1 > 23
2n+1 > 23
henrypickle
2010-02-10 19:48:29
n^2+23<(n+1)^2
n^2+23<(n+1)^2
aopsaccount2131
2010-02-10 19:48:29
(n+1)^2 - n^2 = 2n + !
(n+1)^2 - n^2 = 2n + !
aerrowfinn72
2010-02-10 19:48:50
nxt is 167....so its 167, 23, 6, 3, 2, 1, 0
nxt is 167....so its 167, 23, 6, 3, 2, 1, 0
mgao
2010-02-10 19:48:50
then x = 167.
then x = 167.
rrusczyk
2010-02-10 19:48:57
So 23 + n^2 < (n+1)^2, which means 23 < (n+1)^ - n^2 = 2n + 1. Aha. That gives us n > 11, which means our next term is 23 + 12^2 = 167.
So 23 + n^2 < (n+1)^2, which means 23 < (n+1)^ - n^2 = 2n + 1. Aha. That gives us n > 11, which means our next term is 23 + 12^2 = 167.
rrusczyk
2010-02-10 19:49:03
Finally, we're almost there:
Finally, we're almost there:
rrusczyk
2010-02-10 19:49:08
x, 167, 23, 7, 3, 2, 1, 0.
x, 167, 23, 7, 3, 2, 1, 0.
rrusczyk
2010-02-10 19:49:19
How do we take that next step?
How do we take that next step?
zheng
2010-02-10 19:49:52
Same procedure
Same procedure
henrypickle
2010-02-10 19:49:54
x=n^2+167
x=n^2+167
pumpkinpi
2010-02-10 19:49:54
We do it again: 2n + 1 > 167
We do it again: 2n + 1 > 167
darkdieuguerre
2010-02-10 19:49:54
repeat for 167
repeat for 167
The Archer
2010-02-10 19:49:54
2n+1>167
2n+1>167
Just_Beginner
2010-02-10 19:49:55
same thing
same thing
The Hobbit
2010-02-10 19:49:55
same thing?
same thing?
y2ucc
2010-02-10 19:49:57
same way as before
same way as before
rrusczyk
2010-02-10 19:50:06
Just as before, we want (n+1)^2 > x = 167 + n^2, so 167 < (n+1)^2 - n^2 = 2n + 1, which means n > 83.
Just as before, we want (n+1)^2 > x = 167 + n^2, so 167 < (n+1)^2 - n^2 = 2n + 1, which means n > 83.
karatemagic7
2010-02-10 19:50:24
then n=84, so 84^2+167 mod 10 = 3
then n=84, so 84^2+167 mod 10 = 3
dolk
2010-02-10 19:50:24
((167+1)/2)^2+167
((167+1)/2)^2+167
basketball9
2010-02-10 19:50:24
2n+1>167 so n is 84
2n+1>167 so n is 84
mgao
2010-02-10 19:50:24
then 84^2 + 167 will have units digit 3
then 84^2 + 167 will have units digit 3
TheWorstPlayer
2010-02-10 19:50:27
n = 84, 84^2 + 67 gives us 3 as the last digit
n = 84, 84^2 + 67 gives us 3 as the last digit
rrusczyk
2010-02-10 19:50:35
Therefore, x = 167 + 84^2, which has last digit 3.
Therefore, x = 167 + 84^2, which has last digit 3.
bunnygirl
2010-02-10 19:50:43
can you solve this problem without working backwards?
can you solve this problem without working backwards?
rrusczyk
2010-02-10 19:50:50
I doubt it!
I doubt it!
rrusczyk
2010-02-10 19:50:55
Yes, the answer is B
Yes, the answer is B
aopsaccount2131
2010-02-10 19:51:03
how do we know that it is good to minimize earlier on? could there exist a better solution with larger numbers earlier on?
how do we know that it is good to minimize earlier on? could there exist a better solution with larger numbers earlier on?
rrusczyk
2010-02-10 19:51:35
This is a very good question, and for those of you preparing for the USAMO, you should definitely think about how you could prove that what we did is optimal.
This is a very good question, and for those of you preparing for the USAMO, you should definitely think about how you could prove that what we did is optimal.
rrusczyk
2010-02-10 19:51:59
I recommend you post your thoughts on this on the message board -- it's not trivial to write such a proof in a clear, rigorous way.
I recommend you post your thoughts on this on the message board -- it's not trivial to write such a proof in a clear, rigorous way.
rrusczyk
2010-02-10 19:52:27
We'll take a 2-minute break, and then start with the AMC 12.
We'll take a 2-minute break, and then start with the AMC 12.
rrusczyk
2010-02-10 19:52:51
Again, I'll note that there are nearly 300 people here, so we won't be able to answer all your questions!
Again, I'll note that there are nearly 300 people here, so we won't be able to answer all your questions!
rrusczyk
2010-02-10 19:54:25
We'll do the last 5 problems of the AMC 12 now, and then I'll take requests.
We'll do the last 5 problems of the AMC 12 now, and then I'll take requests.
rrusczyk
2010-02-10 19:54:49
I don't have any idea what the index will be this year -- we'll have to wait and see.
I don't have any idea what the index will be this year -- we'll have to wait and see.
rrusczyk
2010-02-10 19:55:33
(No I can't guess :) )
(No I can't guess :) )
rrusczyk
2010-02-10 19:55:48
On to AMC 12
On to AMC 12
rrusczyk
2010-02-10 19:55:49

rrusczyk
2010-02-10 19:56:17

mgao
2010-02-10 19:56:25
it must be tangent to the line because if it werent it would go below, contradiction.
it must be tangent to the line because if it werent it would go below, contradiction.
rrusczyk
2010-02-10 19:56:32
Let the three points of intersections be (p, f(p)), (q, f(q)), and (r, f(r)).
Let the three points of intersections be (p, f(p)), (q, f(q)), and (r, f(r)).
rrusczyk
2010-02-10 19:56:37

rrusczyk
2010-02-10 19:56:45
Now what?
Now what?
33286
2010-02-10 19:56:56
The line is tangent at the three points. Subtract bx+c from the equation.
The line is tangent at the three points. Subtract bx+c from the equation.
aleph0
2010-02-10 19:56:56
but f(x)-bx-c must have three multiple roots
but f(x)-bx-c must have three multiple roots
darkdieuguerre
2010-02-10 19:57:02
We have that f(x) - y = [(x-p)(x-q)(x-r)]^2
We have that f(x) - y = [(x-p)(x-q)(x-r)]^2
rrusczyk
2010-02-10 19:57:05
Consider the difference g(x) = f(x) - (bx + c).
Consider the difference g(x) = f(x) - (bx + c).
rrusczyk
2010-02-10 19:57:18
How do we know that this difference is the product of three squares of binomials?
How do we know that this difference is the product of three squares of binomials?
33286
2010-02-10 19:57:53
Otherwise it would change sign at the roots.
Otherwise it would change sign at the roots.
zserf
2010-02-10 19:57:54
If there were cubes, then it would cross the line
If there were cubes, then it would cross the line
pumpkinpi
2010-02-10 19:57:56
for each root r there is a factor (x-r)
for each root r there is a factor (x-r)
rrusczyk
2010-02-10 19:58:05
For each root, we have a factor (x-r).
For each root, we have a factor (x-r).
33286
2010-02-10 19:58:37
Or rather, two factors (x-r).
Or rather, two factors (x-r).
rrusczyk
2010-02-10 19:58:41
True, why?
True, why?
Infrared
2010-02-10 19:59:35
The graph has to stay positive, which implies a double root
The graph has to stay positive, which implies a double root
flare
2010-02-10 19:59:38
because the multiplicity of the roots must be even so it does not cross the point... which the diagram proves?
because the multiplicity of the roots must be even so it does not cross the point... which the diagram proves?
rrusczyk
2010-02-10 19:59:41
But if we only had one such factor then when x goes from just above r to just below r, g(x) would change signs. So, there must be a second factor of x-r to keep the sign positive.
But if we only had one such factor then when x goes from just above r to just below r, g(x) would change signs. So, there must be a second factor of x-r to keep the sign positive.
rrusczyk
2010-02-10 20:00:00
(To prove this rigorously, you'd use calculus, but intuitively, we can see why we must have a double root.)
(To prove this rigorously, you'd use calculus, but intuitively, we can see why we must have a double root.)
megahertz
2010-02-10 20:00:04
how do you know they are double roots and not complex?
how do you know they are double roots and not complex?
rrusczyk
2010-02-10 20:00:18
We know we have 3 roots that correspond to the three tangent points.
We know we have 3 roots that correspond to the three tangent points.
rrusczyk
2010-02-10 20:00:23
3 real roots, that is.
3 real roots, that is.
rrusczyk
2010-02-10 20:00:47
And now we know that each root is a double root (or a root 4 times or 6, or whatever)
And now we know that each root is a double root (or a root 4 times or 6, or whatever)
rrusczyk
2010-02-10 20:01:13
Why do we not have a (x-r)^4 in the factorization of f(x)- (bx+c)?
Why do we not have a (x-r)^4 in the factorization of f(x)- (bx+c)?
SuitcaseAsian
2010-02-10 20:01:21
Well it has to be double since there are only 6 roots
Well it has to be double since there are only 6 roots
LincolnMS nerd herd
2010-02-10 20:01:21
but since it's tangent at 2 points, it's 3 roots twice
but since it's tangent at 2 points, it's 3 roots twice
aleph0
2010-02-10 20:01:21
but it can't be 4 or 6 because there are three of them
but it can't be 4 or 6 because there are three of them
turak
2010-02-10 20:01:21
not 4,6,etc because highest exponent is 6
not 4,6,etc because highest exponent is 6
rrusczyk
2010-02-10 20:01:28
Exactly -- there are 6 total roots.
Exactly -- there are 6 total roots.
33286
2010-02-10 20:01:30
g(x) must be a square of the cubic function y=(x-p)(x-q)(x-r).
g(x) must be a square of the cubic function y=(x-p)(x-q)(x-r).
rrusczyk
2010-02-10 20:01:40
We conclude that the multiplicities of the roots x = p, q, and r are all 2.
We conclude that the multiplicities of the roots x = p, q, and r are all 2.
rrusczyk
2010-02-10 20:01:47
Therefore:
Therefore:
rrusczyk
2010-02-10 20:01:56

rrusczyk
2010-02-10 20:02:11
Now what?
Now what?
Just_Beginner
2010-02-10 20:02:32
expand then set coefficents equal to each other?
expand then set coefficents equal to each other?
mathrat
2010-02-10 20:02:32
expand!!!1
expand!!!1
y2ucc
2010-02-10 20:02:36
expand
expand
rrusczyk
2010-02-10 20:02:49
How might we go about expanding in a sensible way?
How might we go about expanding in a sensible way?
rrusczyk
2010-02-10 20:02:58
Do we really want to multiply that whole thing out?
Do we really want to multiply that whole thing out?
gomath888
2010-02-10 20:03:13
expand cubic
expand cubic
rrusczyk
2010-02-10 20:03:22

rrusczyk
2010-02-10 20:03:43
Expanding this square is a little easier. What do we get?
Expanding this square is a little easier. What do we get?
Infrared
2010-02-10 20:05:04
We don't need to expand it more, we just look at how the coefficients multiply for each case
We don't need to expand it more, we just look at how the coefficients multiply for each case
rrusczyk
2010-02-10 20:05:13
True; what do we get for x^5 on the right?
True; what do we get for x^5 on the right?
aleph0
2010-02-10 20:05:22
2A
2A
ksun48
2010-02-10 20:05:22
2A
2A
nearsymmetric
2010-02-10 20:05:24
A=-5
A=-5
aleph0
2010-02-10 20:05:28
so A=-5
so A=-5
rrusczyk
2010-02-10 20:05:29
We get 2A, so A = -5.
We get 2A, so A = -5.
rrusczyk
2010-02-10 20:05:37
Next up, what about x^4 on the right?
Next up, what about x^4 on the right?
aleph0
2010-02-10 20:06:02
A^2+2B
A^2+2B
33286
2010-02-10 20:06:02
A^2+2B
A^2+2B
megahertz
2010-02-10 20:06:02
A^2+2B?
A^2+2B?
Just_Beginner
2010-02-10 20:06:02
B+A^2+B
B+A^2+B
rrusczyk
2010-02-10 20:06:08
We get A^2 + 2B, and A=-5, so B = 2
We get A^2 + 2B, and A=-5, so B = 2
rrusczyk
2010-02-10 20:06:20
And C?
And C?
rrusczyk
2010-02-10 20:06:34
How do we get that?
How do we get that?
aggarwal
2010-02-10 20:06:49
8
8
sparkle123
2010-02-10 20:06:49
2C+2AB
2C+2AB
ksun48
2010-02-10 20:06:49
do x^3
do x^3
darkdieuguerre
2010-02-10 20:06:49
solving for the x^3 term
solving for the x^3 term
aleph0
2010-02-10 20:06:49
2AB+2C=-4?
2AB+2C=-4?
33286
2010-02-10 20:06:49
-4=2AB+2C
-4=2AB+2C
rrusczyk
2010-02-10 20:07:06
The x^3 term gives -4 = 2AB + 2C, and using our values of A and B give C = 8.
The x^3 term gives -4 = 2AB + 2C, and using our values of A and B give C = 8.
rrusczyk
2010-02-10 20:07:11

rrusczyk
2010-02-10 20:07:17
Now what?
Now what?
mgao
2010-02-10 20:07:52
roots can only be factors of 8 and 8 doesnt work so 4 has to be the largest root
roots can only be factors of 8 and 8 doesnt work so 4 has to be the largest root
karatemagic7
2010-02-10 20:07:52
Yay, noq we get that the solutions are -1,2,4.
Yay, noq we get that the solutions are -1,2,4.
CountdownKing
2010-02-10 20:08:08
well -1 works, so factoring out x+1 we will get 2 and 4, so hopefully after coming this far we can realize that 4 is greatest
well -1 works, so factoring out x+1 we will get 2 and 4, so hopefully after coming this far we can realize that 4 is greatest
ytrewq
2010-02-10 20:08:10
factors as (x-4)(x-2)(x+1)
factors as (x-4)(x-2)(x+1)
rrusczyk
2010-02-10 20:08:27
We can factor this cubic as (x+1)(x-2)(x+4) (-1 is obviously a root, and then you have a quadratic after you factor out x+1)
We can factor this cubic as (x+1)(x-2)(x+4) (-1 is obviously a root, and then you have a quadratic after you factor out x+1)
nikeballa96
2010-02-10 20:08:32
another A. good thing we're on another test. :)
another A. good thing we're on another test. :)
33286
2010-02-10 20:08:34
Or we could just plug in all the answer choices XD
Or we could just plug in all the answer choices XD
rrusczyk
2010-02-10 20:08:39
That works too :)
That works too :)
rrusczyk
2010-02-10 20:08:40
Therefore, the largest root is 4, and the answer is (A).
Therefore, the largest root is 4, and the answer is (A).
rrusczyk
2010-02-10 20:08:54
On to:
On to:
rrusczyk
2010-02-10 20:08:54

rrusczyk
2010-02-10 20:09:08
Any suggestions where to start?
Any suggestions where to start?
Poincare
2010-02-10 20:09:16
OMG
OMG
rrusczyk
2010-02-10 20:09:22
Not a bad first thought.
Not a bad first thought.
rrusczyk
2010-02-10 20:09:42
What tool is helpful in dealing with funky functions?
What tool is helpful in dealing with funky functions?
THECHAMP
2010-02-10 20:10:00
graph
graph
nearsymmetric
2010-02-10 20:10:05
graph it
graph it
rrusczyk
2010-02-10 20:10:15

rrusczyk
2010-02-10 20:10:20
What does the graph look like when x is really large?
What does the graph look like when x is really large?
mgao
2010-02-10 20:10:52
slope is huge
slope is huge
joeislittle
2010-02-10 20:10:52
it is a line
it is a line
happyof
2010-02-10 20:10:52
linear very steep
linear very steep
darkdieuguerre
2010-02-10 20:10:56
a straight line
a straight line
Infrared
2010-02-10 20:10:56
7140X-119
7140X-119
zserf
2010-02-10 20:11:01
You can just ignore absolute values in that case.
You can just ignore absolute values in that case.
rrusczyk
2010-02-10 20:11:11
Exactly. It's a line. With slope 1+2+3+...+119
Exactly. It's a line. With slope 1+2+3+...+119
rrusczyk
2010-02-10 20:11:16
What about when x is negative?
What about when x is negative?
ytrewq
2010-02-10 20:12:02
-7140X+119
-7140X+119
dragoneye776
2010-02-10 20:12:02
Slope is negatively steep
Slope is negatively steep
flare
2010-02-10 20:12:02
the graph goes the opposite way?/
the graph goes the opposite way?/
PowerOfPi
2010-02-10 20:12:02
-7140x+119
-7140x+119
natiator
2010-02-10 20:12:02
it's another line
it's another line
nearsymmetric
2010-02-10 20:12:02
opposite slope
opposite slope
rrusczyk
2010-02-10 20:12:06
When x is negative, the graph is a line with slope -(1+2+3+4+ . . . + 119).
When x is negative, the graph is a line with slope -(1+2+3+4+ . . . + 119).
rrusczyk
2010-02-10 20:12:14
What happens in-between?
What happens in-between?
mgao
2010-02-10 20:12:53
the graph looks like a deformed approximation of a parabola and we want to find the "vertex"-ish point
the graph looks like a deformed approximation of a parabola and we want to find the "vertex"-ish point
joeislittle
2010-02-10 20:12:53
lots of lines
lots of lines
mgao
2010-02-10 20:12:53
slope slowly goes up til it reaches 7140
slope slowly goes up til it reaches 7140
MrBob
2010-02-10 20:12:53
intermediate slope
intermediate slope
prezcoin
2010-02-10 20:12:53
it curves down, then back up
it curves down, then back up
Wikipedian1337
2010-02-10 20:12:53
signs start shifting
signs start shifting
nikeballa96
2010-02-10 20:12:53
at some point...it turns around.
at some point...it turns around.
MeowmixOX;D
2010-02-10 20:12:53
less steep slope
less steep slope
nearsymmetric
2010-02-10 20:12:53
we lose some negative slopes, gain some positives
we lose some negative slopes, gain some positives
kthxbai
2010-02-10 20:12:53
shifts from negative to positive every time it hits 1, 1/2, 1/3, 1/4 ...
shifts from negative to positive every time it hits 1, 1/2, 1/3, 1/4 ...
rrusczyk
2010-02-10 20:13:06
What is the smallest value of x at which the slope stops being -(1+2+3+4+ . . . + 119)?
What is the smallest value of x at which the slope stops being -(1+2+3+4+ . . . + 119)?
mgao
2010-02-10 20:13:25
1/119
1/119
aleph0
2010-02-10 20:13:25
1/119
1/119
prezcoin
2010-02-10 20:13:25
1/119
1/119
bbgun34
2010-02-10 20:13:25
x=1/119
x=1/119
darkdieuguerre
2010-02-10 20:13:25
x = 1/119
x = 1/119
rrusczyk
2010-02-10 20:13:31
When x = 1/119, the term |119x - 1| is 0. Then, when x > 1/119, we have |119x - 1| = 119x - 1, instead of -(119x - 1). So, when 1/118 > x > 1/119, the graph is a line with slope -(1+2+3+ . . . + 118) + 119.
When x = 1/119, the term |119x - 1| is 0. Then, when x > 1/119, we have |119x - 1| = 119x - 1, instead of -(119x - 1). So, when 1/118 > x > 1/119, the graph is a line with slope -(1+2+3+ . . . + 118) + 119.
rrusczyk
2010-02-10 20:13:39
Aha, the slope is a little less negative.
Aha, the slope is a little less negative.
nikeballa96
2010-02-10 20:13:52
...still pretty dang negative.
...still pretty dang negative.
rrusczyk
2010-02-10 20:13:58
Indeed. But...
Indeed. But...
mgao
2010-02-10 20:14:00
then 1/118, etc.
then 1/118, etc.
rrusczyk
2010-02-10 20:14:05
As x increases from 0 to 1, the graph is a series of line segments. As x passes each value of the form 1/n, where n is an integer, the corresponding n term in the slope expression switches from positive to negative. For example, the slope when 1/49 > x > 1/50 is -(1+2+3+4+...+49) + (50+51 +. . . 119).
As x increases from 0 to 1, the graph is a series of line segments. As x passes each value of the form 1/n, where n is an integer, the corresponding n term in the slope expression switches from positive to negative. For example, the slope when 1/49 > x > 1/50 is -(1+2+3+4+...+49) + (50+51 +. . . 119).
aleph0
2010-02-10 20:14:25
so when the slope is 0 (or close to it) is when the graph will be at a minimum
so when the slope is 0 (or close to it) is when the graph will be at a minimum
nearsymmetric
2010-02-10 20:14:25
we need the place where the sum of the negatively sloped pieces is equal (or nearly equal) to the sum of the poisitives
we need the place where the sum of the negatively sloped pieces is equal (or nearly equal) to the sum of the poisitives
rrusczyk
2010-02-10 20:14:34
Our problem now is to determine at what value of x the slope switches from negative to positive---when that happens, the graph stops going down and starts going up, and we can find our minimum.
Our problem now is to determine at what value of x the slope switches from negative to positive---when that happens, the graph stops going down and starts going up, and we can find our minimum.
rrusczyk
2010-02-10 20:14:44
How will we find where the sign switches?
How will we find where the sign switches?
rrusczyk
2010-02-10 20:15:20
What should we compute first?
What should we compute first?
dday25
2010-02-10 20:15:45
1+2+3+...+119?
1+2+3+...+119?
sjiang
2010-02-10 20:15:45
1+2+...+119
1+2+...+119
darkdieuguerre
2010-02-10 20:15:45
a smaller triangular sum which is roughly half 119*120/2
a smaller triangular sum which is roughly half 119*120/2
aleph0
2010-02-10 20:15:45
the value 1+2+...+119
the value 1+2+...+119
Wikipedian1337
2010-02-10 20:15:48
2a(a+1)/2 = 119*120/2
2a(a+1)/2 = 119*120/2
rrusczyk
2010-02-10 20:15:51
We start with the sum 1+2+3+. . . + 119 = 119(120)/2 = 119*60 = 7140.
We start with the sum 1+2+3+. . . + 119 = 119(120)/2 = 119*60 = 7140.
rrusczyk
2010-02-10 20:16:05
Then what do want to find?
Then what do want to find?
mgao
2010-02-10 20:16:15
let 1/k be where it switches then we need 1+2+...+k > (k+1) + ... + 119
let 1/k be where it switches then we need 1+2+...+k > (k+1) + ... + 119
zserf
2010-02-10 20:16:30
A triangular number that is close to half of it.
A triangular number that is close to half of it.
aleph0
2010-02-10 20:16:30
another number so that n(n+1) is approximately 7140
another number so that n(n+1) is approximately 7140
rrusczyk
2010-02-10 20:16:32
Then, we want the smallest value of n such that 1+2+...+ n >= 7140/2.
Then, we want the smallest value of n such that 1+2+...+ n >= 7140/2.
Just_Beginner
2010-02-10 20:16:34
a triangluar number about 1/2 that, or 3570
a triangluar number about 1/2 that, or 3570
aleph0
2010-02-10 20:16:52
but this equals 84*85
but this equals 84*85
CountdownKing
2010-02-10 20:16:52
looks like we can be exact with 84
looks like we can be exact with 84
nearsymmetric
2010-02-10 20:16:52
Find n such that 1+2+...+n<=(1/2)*S <=1+2+...+(n+1). where S = 1+2+...+119
Find n such that 1+2+...+n<=(1/2)*S <=1+2+...+(n+1). where S = 1+2+...+119
aleph0
2010-02-10 20:16:52
we can find that 85*84=7140
we can find that 85*84=7140
karanbir
2010-02-10 20:16:56
halfway point is 3570, and it will switch when 1+2+3+...+n-1+n=3570
halfway point is 3570, and it will switch when 1+2+3+...+n-1+n=3570
rrusczyk
2010-02-10 20:17:01
This gives us n(n+1)/2 >= 3570. From here we can do a little trial-and-error to solve n^2 + n >= 7140. We find that we get equality at n=84. What does that tell us?
This gives us n(n+1)/2 >= 3570. From here we can do a little trial-and-error to solve n^2 + n >= 7140. We find that we get equality at n=84. What does that tell us?
darkdieuguerre
2010-02-10 20:17:19
If we evaluate at x = 1/84, we get our minimum
If we evaluate at x = 1/84, we get our minimum
aleph0
2010-02-10 20:17:20
that the segment is flat when 1/85<x<1/84
that the segment is flat when 1/85<x<1/84
rrusczyk
2010-02-10 20:17:29
That tells us that the graph for 1/84 > x > 1/85 is a horizontal line segment. The graph goes upward both to the left and the right of this segment, so the y-coordinate of points on this segment give us our desired minimum. And what is that minimum?
That tells us that the graph for 1/84 > x > 1/85 is a horizontal line segment. The graph goes upward both to the left and the right of this segment, so the y-coordinate of points on this segment give us our desired minimum. And what is that minimum?
darkdieuguerre
2010-02-10 20:17:49
49
49
CountdownKing
2010-02-10 20:17:49
try x=1/84 you will see that we get 49, and we know that our method was optimal because we got the lowest choice
try x=1/84 you will see that we get 49, and we know that our method was optimal because we got the lowest choice
mathrat
2010-02-10 20:17:49
49
49
aleph0
2010-02-10 20:17:59
84-(119-84)
84-(119-84)
33286
2010-02-10 20:17:59
84-(119-84)=49
84-(119-84)=49
rrusczyk
2010-02-10 20:18:03

rrusczyk
2010-02-10 20:18:09
All terms of the form |nx - 1| for n >= 85 equal nx - 1 for x > 1/85, and all terms of the form |nx-1| for n <= 84 equal 1 - nx for x < 1/84.
All terms of the form |nx - 1| for n >= 85 equal nx - 1 for x > 1/85, and all terms of the form |nx-1| for n <= 84 equal 1 - nx for x < 1/84.
rrusczyk
2010-02-10 20:18:14

rrusczyk
2010-02-10 20:18:22
We just saw that 1+2+3+ . . . + 84 = 85 + 86 + . . . + 119, so all the nx terms cancel out when 1/84 > x > 1/85. We're left with 84 terms of +1 and (119-84) terms of (-1), so our minimum is 84 - 35 = 49.
We just saw that 1+2+3+ . . . + 84 = 85 + 86 + . . . + 119, so all the nx terms cancel out when 1/84 > x > 1/85. We're left with 84 terms of +1 and (119-84) terms of (-1), so our minimum is 84 - 35 = 49.
EricMathPath09
2010-02-10 20:18:25
ANOTHER A?!
ANOTHER A?!
nikeballa96
2010-02-10 20:18:25
ANOTHER A?!
ANOTHER A?!
rrusczyk
2010-02-10 20:18:32
I hear an echo.
I hear an echo.
rrusczyk
2010-02-10 20:18:54
Fortunately, we've already taken care of #23; it was the same as #24 on the AMC 10
Fortunately, we've already taken care of #23; it was the same as #24 on the AMC 10
rrusczyk
2010-02-10 20:19:05
(And it was another A, for those of you keeping score at home)
(And it was another A, for those of you keeping score at home)
megahertz
2010-02-10 20:19:18
Are they trying to mess with us? Next year they will switch it to E and millions of children will guess the last 5 wrong.
Are they trying to mess with us? Next year they will switch it to E and millions of children will guess the last 5 wrong.
rrusczyk
2010-02-10 20:19:21
True
True
rrusczyk
2010-02-10 20:19:28

rrusczyk
2010-02-10 20:19:34
The domain of f(x) is the set of x for which f(x) is defined. For which x is f(x) defined?
The domain of f(x) is the set of x for which f(x) is defined. For which x is f(x) defined?
darkdieuguerre
2010-02-10 20:19:57
when the product of the sine functions is strictly positive
when the product of the sine functions is strictly positive
aleph0
2010-02-10 20:19:57
when that product is positive
when that product is positive
gomath888
2010-02-10 20:19:57
the product of sines is positive
the product of sines is positive
Wikipedian1337
2010-02-10 20:19:57
whenever the thing in the log > 0
whenever the thing in the log > 0
rrusczyk
2010-02-10 20:20:05

rrusczyk
2010-02-10 20:20:09

Wikipedian1337
2010-02-10 20:20:39
at a/n where a is an integer
at a/n where a is an integer
33286
2010-02-10 20:20:39
k/n
k/n
GoldenFrog1618
2010-02-10 20:20:39
x is a multiple of 1/n
x is a multiple of 1/n
rrusczyk
2010-02-10 20:20:47

rrusczyk
2010-02-10 20:21:08
Now, how might we organize information about when each factor is positive or negative?
Now, how might we organize information about when each factor is positive or negative?
aleph0
2010-02-10 20:21:30
draw a diagram?
draw a diagram?
Just_Beginner
2010-02-10 20:21:30
make a chart
make a chart
darkdieuguerre
2010-02-10 20:21:30
the sine function alternates between positive and negative after each zero
the sine function alternates between positive and negative after each zero
rrusczyk
2010-02-10 20:22:03
We can make a sign chart to keep track of where each switches sign.
We can make a sign chart to keep track of where each switches sign.
rrusczyk
2010-02-10 20:22:13

rrusczyk
2010-02-10 20:22:17

rrusczyk
2010-02-10 20:22:26
In the same way, we can construct the sign chart for each of the functions sin(pi x), sin(2 pi x), ..., sin(8 pi x).
In the same way, we can construct the sign chart for each of the functions sin(pi x), sin(2 pi x), ..., sin(8 pi x).
rrusczyk
2010-02-10 20:22:40

aleph0
2010-02-10 20:22:45
(or a sine chart :) )
(or a sine chart :) )
Poincare
2010-02-10 20:22:54
the chart? how do you read it?
the chart? how do you read it?
rrusczyk
2010-02-10 20:23:08
Each number line represents a single factor.
Each number line represents a single factor.
rrusczyk
2010-02-10 20:23:14
The first is sin(pi*x).
The first is sin(pi*x).
rrusczyk
2010-02-10 20:23:19
The number line goes from 0 to 1.
The number line goes from 0 to 1.
rrusczyk
2010-02-10 20:23:39
We put a + where the factor is positive, a 0 where it is 0, and a - where it is negative.
We put a + where the factor is positive, a 0 where it is 0, and a - where it is negative.
rrusczyk
2010-02-10 20:23:43
http://s3.amazonaws.com/classroom.artofproblemsolving.com/Images/Transcripts/811a277df64c0aaa792f534aac1d18dc.png
http://s3.amazonaws.com/classroom.artofproblemsolving.com/Images/Transcripts/811a277df64c0aaa792f534aac1d18dc.png
rrusczyk
2010-02-10 20:23:49
There's a link for the image.
There's a link for the image.
Poincare
2010-02-10 20:23:51
oh okay. so, at 1/3 sin (3pi x) is 0 right?
oh okay. so, at 1/3 sin (3pi x) is 0 right?
rrusczyk
2010-02-10 20:23:54
Right.
Right.
nikeballa96
2010-02-10 20:23:58
that's kinda awesome.
that's kinda awesome.
rrusczyk
2010-02-10 20:24:05
Ya, a nice way to track information.
Ya, a nice way to track information.
mgao
2010-02-10 20:24:12
so somehow, we need to find the number of fragments that that splits it into.
so somehow, we need to find the number of fragments that that splits it into.
darkdieuguerre
2010-02-10 20:24:17
so now the only problem is keeping track of numbers where multiple functions have zeroes
so now the only problem is keeping track of numbers where multiple functions have zeroes
aleph0
2010-02-10 20:24:19
so at any value of x draw a vertical line at x, if 0 is on that line then it is zero, otherwise if there are an odd number of -s then it is negative and an even number makes it positive
so at any value of x draw a vertical line at x, if 0 is on that line then it is zero, otherwise if there are an odd number of -s then it is negative and an even number makes it positive
rrusczyk
2010-02-10 20:24:34
Exactly. We'll have to be pretty organized about this.
Exactly. We'll have to be pretty organized about this.
rrusczyk
2010-02-10 20:24:54
How will we get organized? What should we do?
How will we get organized? What should we do?
aleph0
2010-02-10 20:25:25
we can see that there are two lined up zeroes at 1/4, 1/3, 1/2, 2/3, and 3,4 (4 in the case of 1/2)
we can see that there are two lined up zeroes at 1/4, 1/3, 1/2, 2/3, and 3,4 (4 in the case of 1/2)
rrusczyk
2010-02-10 20:25:41
Yeah, we're going to have to worry about points at which two factors switch.
Yeah, we're going to have to worry about points at which two factors switch.
rrusczyk
2010-02-10 20:25:58
In order to count the intervals, what can we count?
In order to count the intervals, what can we count?
darkdieuguerre
2010-02-10 20:26:11
number of distinct zeroes
number of distinct zeroes
yforman
2010-02-10 20:26:18
the total number of vertical lines excluding the ones aleph0 listed
the total number of vertical lines excluding the ones aleph0 listed
rrusczyk
2010-02-10 20:26:23
We can count endpoints of our intervals.
We can count endpoints of our intervals.
aleph0
2010-02-10 20:26:40
but if two intervals share an endpoint?
but if two intervals share an endpoint?
rrusczyk
2010-02-10 20:26:47
Indeed -- we'll have to be careful about that.
Indeed -- we'll have to be careful about that.
rrusczyk
2010-02-10 20:26:53
Let's start dumb, and then fix it.
Let's start dumb, and then fix it.
rrusczyk
2010-02-10 20:27:11
How many total 0's (endpoints) do we have if we don't care about the ones that share?
How many total 0's (endpoints) do we have if we don't care about the ones that share?
rrusczyk
2010-02-10 20:27:49
(I'm not looking for distinct 0's; I'm looking for a quick count of total 0s)
(I'm not looking for distinct 0's; I'm looking for a quick count of total 0s)
rrusczyk
2010-02-10 20:27:54

yforman
2010-02-10 20:28:03
2+3+...+9 = 44
2+3+...+9 = 44
c32r17qp181cd912
2010-02-10 20:28:10
44.
44.
simplyMathLete
2010-02-10 20:28:10
44
44
Iggy Iguana
2010-02-10 20:28:18
44
44
briantix
2010-02-10 20:28:18
44
44
rrusczyk
2010-02-10 20:28:22
We have n + 1 endpoints (0s) for each n, 1 <= n <= 8, so we get 2 + 3 + 4 + ... + 9 = 44 endpoints.
We have n + 1 endpoints (0s) for each n, 1 <= n <= 8, so we get 2 + 3 + 4 + ... + 9 = 44 endpoints.
rrusczyk
2010-02-10 20:28:29
But clearly, some endpoints are double counted, or more. Which endpoints are counted more than once?
But clearly, some endpoints are double counted, or more. Which endpoints are counted more than once?
darkdieuguerre
2010-02-10 20:29:08
x = 0 and x=1
x = 0 and x=1
c32r17qp181cd912
2010-02-10 20:29:08
The end zeroes.
The end zeroes.
ytrewq
2010-02-10 20:29:08
0, 1/4, 1/3, 1/2, 2/3, 3/4, 1
0, 1/4, 1/3, 1/2, 2/3, 3/4, 1
ksun48
2010-02-10 20:29:08
0, 1/4, 1/3, 1/4, 2/3, 3/4, 1
0, 1/4, 1/3, 1/4, 2/3, 3/4, 1
aleph0
2010-02-10 20:29:08
1/4,1/3,1/2,2/3,3/4
1/4,1/3,1/2,2/3,3/4
CountdownKing
2010-02-10 20:29:08
1/2,1/3,1/4,2/3,3/4
1/2,1/3,1/4,2/3,3/4
aleph0
2010-02-10 20:29:08
and also 0 and 1
and also 0 and 1
rrusczyk
2010-02-10 20:29:17
The quarters, halves, thirds, and the ends.
The quarters, halves, thirds, and the ends.
rrusczyk
2010-02-10 20:29:28
How do we correct for these?
How do we correct for these?
pirox
2010-02-10 20:29:56
subtract
subtract
gh625
2010-02-10 20:29:57
subtract
subtract
aleph0
2010-02-10 20:29:57
subtract them?
subtract them?
bbgun34
2010-02-10 20:29:57
subtract the repeats
subtract the repeats
zserf
2010-02-10 20:29:57
subtract the duplicates.
subtract the duplicates.
rrusczyk
2010-02-10 20:30:08
OK, let's deal with the ends first. What do we have to subtract for these?
OK, let's deal with the ends first. What do we have to subtract for these?
aleph0
2010-02-10 20:30:34
7 for each end
7 for each end
bbgun34
2010-02-10 20:30:34
7 and 7; 14 in all
7 and 7; 14 in all
pirox
2010-02-10 20:30:34
44-14
44-14
yforman
2010-02-10 20:30:34
7 each = 14 total
7 each = 14 total
tenniskidperson3
2010-02-10 20:30:34
7 each
7 each
ytrewq
2010-02-10 20:30:34
subtract 7 each
subtract 7 each
FantasyLover
2010-02-10 20:30:36
7*2
7*2
rrusczyk
2010-02-10 20:30:39
Each end is counted 8 times, so we have to subtract each 7 times.
Each end is counted 8 times, so we have to subtract each 7 times.
rrusczyk
2010-02-10 20:30:59
OK, hou about the halves, quarters, and thirds?
OK, hou about the halves, quarters, and thirds?
ytrewq
2010-02-10 20:31:58
3 for the halves, 1 for all the others
3 for the halves, 1 for all the others
pumpkinpi
2010-02-10 20:31:58
1/2 is counted four times, so we subtract 3: 30-3=27. 1/4, 1/3, 2/3, and 3/4 are each counted twice, so we subtract 4: 27-4=23
1/2 is counted four times, so we subtract 3: 30-3=27. 1/4, 1/3, 2/3, and 3/4 are each counted twice, so we subtract 4: 27-4=23
kthxbai
2010-02-10 20:31:58
1/2 is counted 4 times , so subtract 3. 1/4 and 3/4 are each counted twice, so subtract 1*2=2. 1/3 and 2/3 are each counted twice, so subtract 1*2=2.
1/2 is counted 4 times , so subtract 3. 1/4 and 3/4 are each counted twice, so subtract 1*2=2. 1/3 and 2/3 are each counted twice, so subtract 1*2=2.
rrusczyk
2010-02-10 20:32:05
The endpoint 1/2 is counted 4 times. The endpoints 1/4 and 3/4 are counted twice. The endpoints 1/3 and 2/3 are counted twice.
The endpoint 1/2 is counted 4 times. The endpoints 1/4 and 3/4 are counted twice. The endpoints 1/3 and 2/3 are counted twice.
pumpkinpi
2010-02-10 20:32:07
thus we have 23 endpoints and 22 intervals -> D
thus we have 23 endpoints and 22 intervals -> D
rrusczyk
2010-02-10 20:32:14
Hence, the total number of endpoints is 44 - 2 x 7 - 3 - 2 x 1 - 2 x 1 = 23, which means there are 22 open sub-intervals.
Hence, the total number of endpoints is 44 - 2 x 7 - 3 - 2 x 1 - 2 x 1 = 23, which means there are 22 open sub-intervals.
rrusczyk
2010-02-10 20:32:19
As x increases from 0 to 1, x will hit these endpoints. At these values, does g(x) always change sign, from positive to negative or negative to positive?
As x increases from 0 to 1, x will hit these endpoints. At these values, does g(x) always change sign, from positive to negative or negative to positive?
ytrewq
2010-02-10 20:32:42
not at the double counted ones
not at the double counted ones
33286
2010-02-10 20:32:42
No - at 1/3, 1/2, 1/4, 3/4 and 2/3 it doesn't
No - at 1/3, 1/2, 1/4, 3/4 and 2/3 it doesn't
c32r17qp181cd912
2010-02-10 20:32:42
Not always.
Not always.
rrusczyk
2010-02-10 20:32:49
No, because an even number of factors may change sign simultaneously. For example, as x passes through 1/4, both sin(4 pi x) and sin(8 pi x) change sign. As a result, g(x) is positive for x slightly less than 1/4, and x slightly greater than 1/4.
No, because an even number of factors may change sign simultaneously. For example, as x passes through 1/4, both sin(4 pi x) and sin(8 pi x) change sign. As a result, g(x) is positive for x slightly less than 1/4, and x slightly greater than 1/4.
rrusczyk
2010-02-10 20:32:56
The values of x for which an even number of factors change simultaneously are the values of x which appear as an endpoint in an even number of sign charts. These are exactly the endpoints that were double counted above.
The values of x for which an even number of factors change simultaneously are the values of x which appear as an endpoint in an even number of sign charts. These are exactly the endpoints that were double counted above.
rrusczyk
2010-02-10 20:33:05
Now we know how the sign of g(x) behaves, we can construct the sign chart of g(x).
Now we know how the sign of g(x) behaves, we can construct the sign chart of g(x).
rrusczyk
2010-02-10 20:33:13

rrusczyk
2010-02-10 20:33:30
What do we do now?
What do we do now?
rrusczyk
2010-02-10 20:33:46
The sign of g(x) alternates at each endpoint (that is, it goes from positive to negative or negative to positive), except at the endpoints we indicated above, namely 1/2, 1/4, 3/4, 1/3, and 2/3.
The sign of g(x) alternates at each endpoint (that is, it goes from positive to negative or negative to positive), except at the endpoints we indicated above, namely 1/2, 1/4, 3/4, 1/3, and 2/3.
aleph0
2010-02-10 20:33:49
so then we can count the pluses to get 12, B
so then we can count the pluses to get 12, B
darkdieuguerre
2010-02-10 20:33:49
12 positive regions = B
12 positive regions = B
bobma99
2010-02-10 20:33:49
count
count
yforman
2010-02-10 20:33:49
count the "+"s
count the "+"s
mgao
2010-02-10 20:33:49
count the number of +s
count the number of +s
tenniskidperson3
2010-02-10 20:33:49
The answer is 12
The answer is 12
rrusczyk
2010-02-10 20:33:54
Now we can read off the sign chart the number of open sub-intervals where g(x) is positive, which is 12. The answer is (B).
Now we can read off the sign chart the number of open sub-intervals where g(x) is positive, which is 12. The answer is (B).
rrusczyk
2010-02-10 20:34:03
Pretty tough problem -- that's why it's #24.
Pretty tough problem -- that's why it's #24.
aleph0
2010-02-10 20:34:19
25 is harder
25 is harder
rrusczyk
2010-02-10 20:34:22
LEt's see!
LEt's see!
rrusczyk
2010-02-10 20:34:36

rrusczyk
2010-02-10 20:34:50
Let a, b, c, and d be the sides of the quadrilateral, so a + b + c + d = 32. Is there anything else we can immediately say about these side lengths?
Let a, b, c, and d be the sides of the quadrilateral, so a + b + c + d = 32. Is there anything else we can immediately say about these side lengths?
BarbieRocks
2010-02-10 20:35:12
a+b+c>d?
a+b+c>d?
billy314
2010-02-10 20:35:12
a+b+c < d
a+b+c < d
rrusczyk
2010-02-10 20:35:16
By the triangle inequality, any side length must be less than the sum of the other three side lengths.
By the triangle inequality, any side length must be less than the sum of the other three side lengths.
LincolnMS nerd herd
2010-02-10 20:35:18
a+b+c<d?
a+b+c<d?
rrusczyk
2010-02-10 20:35:24

rrusczyk
2010-02-10 20:35:33
What do these tell us?
What do these tell us?
aleph0
2010-02-10 20:35:47
all <16
all <16
darkdieuguerre
2010-02-10 20:35:47
they cannot be greater than or equal to 16
they cannot be greater than or equal to 16
mgao
2010-02-10 20:35:47
thus, none of them are larger than 15.
thus, none of them are larger than 15.
xpmath
2010-02-10 20:35:47
each of the sides is less than 16
each of the sides is less than 16
rrusczyk
2010-02-10 20:35:52
Let's consider the first inequality, a < b + c + d.
Let's consider the first inequality, a < b + c + d.
rrusczyk
2010-02-10 20:35:59
Since a + b + c + d = 32, the inequality becomes a < 32 - a, so 2a < 32, or a < 16.
Since a + b + c + d = 32, the inequality becomes a < 32 - a, so 2a < 32, or a < 16.
rrusczyk
2010-02-10 20:36:04
Since a is a positive integer, a <= 15. Similarly, b, c, d <= 15.
Since a is a positive integer, a <= 15. Similarly, b, c, d <= 15.
rrusczyk
2010-02-10 20:36:11
Conversely, suppose a <= 15. Then a < 16, which is equivalent to a < b + c + d. Similarly, the other inequalities are satisfied.
Conversely, suppose a <= 15. Then a < 16, which is equivalent to a < b + c + d. Similarly, the other inequalities are satisfied.
rrusczyk
2010-02-10 20:36:19
Therefore, as long as a, b, c, and d are all at most 15, there exists a convex quadrilateral whose side lengths are a, b, c, and d
Therefore, as long as a, b, c, and d are all at most 15, there exists a convex quadrilateral whose side lengths are a, b, c, and d
mgao
2010-02-10 20:36:26
first of all a quadrangle has one degree of freedom so any quadrangle can be cyclic.
first of all a quadrangle has one degree of freedom so any quadrangle can be cyclic.
33286
2010-02-10 20:36:26
However, any sequence of side lengths that is a quadrilateral at all may form a cyclic quadrilateral.
However, any sequence of side lengths that is a quadrilateral at all may form a cyclic quadrilateral.
tenniskidperson3
2010-02-10 20:36:41
There is exactly one cyclic quadrilateral per valid regular quadrilateral with ordered sides.
There is exactly one cyclic quadrilateral per valid regular quadrilateral with ordered sides.
rrusczyk
2010-02-10 20:36:43
Exactly. Furthermore, this cyclic quadrilateral is unique, in the sense given in the problem.
Exactly. Furthermore, this cyclic quadrilateral is unique, in the sense given in the problem.
rrusczyk
2010-02-10 20:36:52
The idea is as follows: Suppose that the quadrilateral is made of links and hinges. Then unlike the triangle, the quadrilateral can flex, and the angles can change, with the side lengths remaining the same.
The idea is as follows: Suppose that the quadrilateral is made of links and hinges. Then unlike the triangle, the quadrilateral can flex, and the angles can change, with the side lengths remaining the same.
rrusczyk
2010-02-10 20:37:05
(Cyclic means that we can inscribe the quadrilateral in a circle.
(Cyclic means that we can inscribe the quadrilateral in a circle.
rrusczyk
2010-02-10 20:37:17
I really think they should have included that in the problem...)
I really think they should have included that in the problem...)
rrusczyk
2010-02-10 20:37:22

rrusczyk
2010-02-10 20:37:31
Consider one pair of opposite vertices. The quadrilateral can be flexed so that at one extreme, the angle at one of these vertices is close to 180 degrees, so that the sum of the angles at both vertices is greater than 180 degrees.
Consider one pair of opposite vertices. The quadrilateral can be flexed so that at one extreme, the angle at one of these vertices is close to 180 degrees, so that the sum of the angles at both vertices is greater than 180 degrees.
rrusczyk
2010-02-10 20:37:38
But then at the other extreme, one of the angles at the other two opposite vertices can be made close to 180 degrees, and the sum of the angles at the other pair of opposite vertices is greater than 180 degrees.
But then at the other extreme, one of the angles at the other two opposite vertices can be made close to 180 degrees, and the sum of the angles at the other pair of opposite vertices is greater than 180 degrees.
rrusczyk
2010-02-10 20:37:46
At exactly one point in between, the sum of the angles of opposite vertices must have been 180 degrees. At this point, the quadrilateral is cyclic.
At exactly one point in between, the sum of the angles of opposite vertices must have been 180 degrees. At this point, the quadrilateral is cyclic.
darkdieuguerre
2010-02-10 20:38:04
is this statement easily provable?
is this statement easily provable?
rrusczyk
2010-02-10 20:38:06
Not "easy" to be completely rigorous.
Not "easy" to be completely rigorous.
Wikipedian1337
2010-02-10 20:38:08
somebody should post a rigorous proof of this on the message board
somebody should post a rigorous proof of this on the message board
rrusczyk
2010-02-10 20:38:12
That's a good idea.
That's a good idea.
simplyMathLete
2010-02-10 20:38:19
so there is exactly one for each set of sides
so there is exactly one for each set of sides
rrusczyk
2010-02-10 20:38:32
Hence, every quadruple (a,b,c,d) that satisfies a + b + c + d = 32 and a <= 15, b <= 15, c <= 15, and d <= 15 corresponds to a unique cyclic quadrilateral.
Hence, every quadruple (a,b,c,d) that satisfies a + b + c + d = 32 and a <= 15, b <= 15, c <= 15, and d <= 15 corresponds to a unique cyclic quadrilateral.
zserf
2010-02-10 20:38:34
So any 4 lengths that make a quadrilateral can be inscribed in a circle?
So any 4 lengths that make a quadrilateral can be inscribed in a circle?
rrusczyk
2010-02-10 20:38:37
correct
correct
tenniskidperson3
2010-02-10 20:38:46
No, the sides have to be ordered.
No, the sides have to be ordered.
rrusczyk
2010-02-10 20:38:52
right; given the order.
right; given the order.
aggarwal
2010-02-10 20:38:54
isnt it a combinatorics problem
isnt it a combinatorics problem
rrusczyk
2010-02-10 20:38:57
It is now.
It is now.
c32r17qp181cd912
2010-02-10 20:39:01
Now all we have to do is find the number of possible side groupings.
Now all we have to do is find the number of possible side groupings.
aleph0
2010-02-10 20:39:01
so now we consider how many ordered sets of sides there are
so now we consider how many ordered sets of sides there are
rrusczyk
2010-02-10 20:39:27
How will we go about this count
How will we go about this count
rrusczyk
2010-02-10 20:39:28
?
?
Iggy Iguana
2010-02-10 20:39:52
complementary counting?
complementary counting?
tenniskidperson3
2010-02-10 20:39:52
Complementary Counting, I suppose
Complementary Counting, I suppose
rrusczyk
2010-02-10 20:39:57
We can use complementary counting. We can count the number of quadruples a + b + c + d = 32, without the restriction that all variables are at most 15, and then discard the quadruples we don't want to count, i.e. those quadruples where some variable is at least 16.
We can use complementary counting. We can count the number of quadruples a + b + c + d = 32, without the restriction that all variables are at most 15, and then discard the quadruples we don't want to count, i.e. those quadruples where some variable is at least 16.
rrusczyk
2010-02-10 20:40:04
How many quadruples of positive integers (a,b,c,d) satisfy a + b + c + d = 32?
How many quadruples of positive integers (a,b,c,d) satisfy a + b + c + d = 32?
xpmath
2010-02-10 20:40:38
31C3
31C3
aggarwal
2010-02-10 20:40:38
4495
4495
mgao
2010-02-10 20:40:38
(31 choose 3)
(31 choose 3)
Just_Beginner
2010-02-10 20:40:38
31C3
31C3
darkdieuguerre
2010-02-10 20:40:38
31C3
31C3
ytrewq
2010-02-10 20:40:38
31C3
31C3
briantix
2010-02-10 20:40:40
31 coose 3
31 coose 3
rrusczyk
2010-02-10 20:40:45
There is a standard technique, sometimes called stars-and-bars, for counting the number of such quadruples.
There is a standard technique, sometimes called stars-and-bars, for counting the number of such quadruples.
rrusczyk
2010-02-10 20:40:49
Imagine a row of 32 stars. We are allowed to insert 3 bars between any two adjacent stars, and there is at most one bar between any two adjacent stars.
Imagine a row of 32 stars. We are allowed to insert 3 bars between any two adjacent stars, and there is at most one bar between any two adjacent stars.
rrusczyk
2010-02-10 20:40:53
For example, if we had ten stars, the bars may be placed like this: *|***|**|****
For example, if we had ten stars, the bars may be placed like this: *|***|**|****
rrusczyk
2010-02-10 20:40:58
This placement of bars then corresponds to the quadruple (1,3,2,4), where each number represents the number of stars in the groups determined by the bars. Note that there are 9 places to place 3 bars.
This placement of bars then corresponds to the quadruple (1,3,2,4), where each number represents the number of stars in the groups determined by the bars. Note that there are 9 places to place 3 bars.
Just_Beginner
2010-02-10 20:41:01
that is 4495
that is 4495
nikeballa96
2010-02-10 20:41:01
which is 4495
which is 4495
rrusczyk
2010-02-10 20:41:08

rrusczyk
2010-02-10 20:41:18
Now we must subtract the number of quadruples where some variable is at least 16. What makes our job easier?
Now we must subtract the number of quadruples where some variable is at least 16. What makes our job easier?
Just_Beginner
2010-02-10 20:41:44
only one can be at ost 16
only one can be at ost 16
ytrewq
2010-02-10 20:41:44
only one can be greater than 16 at one time
only one can be greater than 16 at one time
rrusczyk
2010-02-10 20:41:52
If some variable is at least 16, then it is the only variable that is at least 16, because a, b, c, and d are positive integers satisfying a + b + c + d = 32. Hence, we can treat each variable separately.
If some variable is at least 16, then it is the only variable that is at least 16, because a, b, c, and d are positive integers satisfying a + b + c + d = 32. Hence, we can treat each variable separately.
rrusczyk
2010-02-10 20:41:57
What is the number of quadruples (a,b,c,d) such that a + b + c + d = 32 and a >= 16? How can we compute this?
What is the number of quadruples (a,b,c,d) such that a + b + c + d = 32 and a >= 16? How can we compute this?
mgao
2010-02-10 20:42:11
we just let a' = a-15, if a>15.
we just let a' = a-15, if a>15.
bbgun34
2010-02-10 20:42:12
take the variable at least 16 and subtract 15 from it
take the variable at least 16 and subtract 15 from it
ksun48
2010-02-10 20:42:12
subtract 15 from a
subtract 15 from a
rrusczyk
2010-02-10 20:42:20
To apply the stars-and-bars argument, let t = a - 15. Then t is a positive integer, and t + b + c + d = (a - 15) + b + c + d = 17. So how many such quadruples (t,b,c,d) are there?
To apply the stars-and-bars argument, let t = a - 15. Then t is a positive integer, and t + b + c + d = (a - 15) + b + c + d = 17. So how many such quadruples (t,b,c,d) are there?
mgao
2010-02-10 20:42:53
(16 choose 3)
(16 choose 3)
darkdieuguerre
2010-02-10 20:42:53
16C3
16C3
bbgun34
2010-02-10 20:42:53
16C3
16C3
Just_Beginner
2010-02-10 20:42:53
16C3
16C3
aleph0
2010-02-10 20:42:53
16 choose 3
16 choose 3
mz94
2010-02-10 20:42:53
16c3
16c3
karatemagic7
2010-02-10 20:42:53
16C3
16C3
tenniskidperson3
2010-02-10 20:42:53
16 c 3=560
16 c 3=560
gh625
2010-02-10 20:42:53
16C3
16C3
rrusczyk
2010-02-10 20:43:01

rrusczyk
2010-02-10 20:43:04
Therefore, there are 560 quadruples of positive integers (a,b,c,d) such that a + b + c + d = 32 and a >= 16.
Therefore, there are 560 quadruples of positive integers (a,b,c,d) such that a + b + c + d = 32 and a >= 16.
mgao
2010-02-10 20:43:13
and any variable can be the big one so we have 4(16 choose 3).
and any variable can be the big one so we have 4(16 choose 3).
FantasyLover
2010-02-10 20:43:13
*4 because of the symmetry
*4 because of the symmetry
rrusczyk
2010-02-10 20:43:17
But we have the same number for the cases b >= 16, c >= 16, and d >= 16.
But we have the same number for the cases b >= 16, c >= 16, and d >= 16.
rrusczyk
2010-02-10 20:43:26
Therefore, the number of quadruples of positive integers (a,b,c,d) such that a + b + c + d = 32 and all variables are at most 15 is 4495 - 4 x 560 = 2255.
Therefore, the number of quadruples of positive integers (a,b,c,d) such that a + b + c + d = 32 and all variables are at most 15 is 4495 - 4 x 560 = 2255.
rrusczyk
2010-02-10 20:43:36
Now what?
Now what?
aleph0
2010-02-10 20:43:54
but what about rotations
but what about rotations
Wikipedian1337
2010-02-10 20:43:57
whats considered distinct?
whats considered distinct?
rrusczyk
2010-02-10 20:44:02
Oh yeah....
Oh yeah....
rrusczyk
2010-02-10 20:44:10
We have to worry about rotations
We have to worry about rotations
33286
2010-02-10 20:44:14
we've counted most possibilites 4 times: abcd, bcda, cdab, dabc. But some we've only counted twice and one once.
we've counted most possibilites 4 times: abcd, bcda, cdab, dabc. But some we've only counted twice and one once.
rrusczyk
2010-02-10 20:44:19
Now, since we are allowed to rotate the quadrilateral, the four quadruples (a,b,c,d), (b,c,d,a), (c,d,a,b), and (d,a,b,c) all represent the same cyclic quadrilateral. (However, (d,c,b,a) does not necessarily represent the same cyclic quadrilateral, since we are not allowed to reflect the quadrilateral.)
Now, since we are allowed to rotate the quadrilateral, the four quadruples (a,b,c,d), (b,c,d,a), (c,d,a,b), and (d,a,b,c) all represent the same cyclic quadrilateral. (However, (d,c,b,a) does not necessarily represent the same cyclic quadrilateral, since we are not allowed to reflect the quadrilateral.)
rrusczyk
2010-02-10 20:44:24
So it seems that each cyclic quadrilateral we are interested in corresponds to four quadruples in our set, so we should be able to simply divide 2255 by 4 to get the answer. But 2255 is not divisible by 4. Have we made a mistake?
So it seems that each cyclic quadrilateral we are interested in corresponds to four quadruples in our set, so we should be able to simply divide 2255 by 4 to get the answer. But 2255 is not divisible by 4. Have we made a mistake?
darkdieuguerre
2010-02-10 20:44:52
because we have quadruples like 8,8,8,8
because we have quadruples like 8,8,8,8
aleph0
2010-02-10 20:44:52
some of them (like 8,8,8,8) will not be affected
some of them (like 8,8,8,8) will not be affected
karatemagic7
2010-02-10 20:44:52
No becasue 8888 is only counted once.
No becasue 8888 is only counted once.
ksun48
2010-02-10 20:44:52
but what about (8,8,8,8)?
but what about (8,8,8,8)?
Visitor03
2010-02-10 20:45:07
rrusczyk, how do you type so fast?
rrusczyk, how do you type so fast?
rrusczyk
2010-02-10 20:45:09
25 fingers. Word's with z give me problems.
25 fingers. Word's with z give me problems.
rrusczyk
2010-02-10 20:45:16
Not every cyclic quadrilateral corresponds to four quadruples in our set, because not every quadruple (a,b,c,d) generates four different quadruples. For example, for a = b = c = d = 8, we only get one distinct quadruple, namely (8,8,8,8), and for a = c = 1 and b = d = 15, we only get two distinct quadruples, namely (1,15,1,15) and (15,1,15,1). So we have to account for possible repetitions.
Not every cyclic quadrilateral corresponds to four quadruples in our set, because not every quadruple (a,b,c,d) generates four different quadruples. For example, for a = b = c = d = 8, we only get one distinct quadruple, namely (8,8,8,8), and for a = c = 1 and b = d = 15, we only get two distinct quadruples, namely (1,15,1,15) and (15,1,15,1). So we have to account for possible repetitions.
rrusczyk
2010-02-10 20:45:27
So we consider the four quadruples (a,b,c,d), (b,c,d,a), (c,d,a,b), and (d,a,b,c), and the ways they can be equal to each other.
So we consider the four quadruples (a,b,c,d), (b,c,d,a), (c,d,a,b), and (d,a,b,c), and the ways they can be equal to each other.
rrusczyk
2010-02-10 20:45:36
Suppose the first two quadruples (a,b,c,d) and (b,c,d,a) are equal. What does this say?
Suppose the first two quadruples (a,b,c,d) and (b,c,d,a) are equal. What does this say?
tenniskidperson3
2010-02-10 20:46:00
a=b=c=d=8
a=b=c=d=8
darkdieuguerre
2010-02-10 20:46:00
a=b=c=d
a=b=c=d
aleph0
2010-02-10 20:46:00
a=b, b=c, c=d, d=a
a=b, b=c, c=d, d=a
yforman
2010-02-10 20:46:02
a=b=c=d=8, and all four quadruples are equal
a=b=c=d=8, and all four quadruples are equal
rrusczyk
2010-02-10 20:46:04
If (a,b,c,d) and (b,c,d,a) are equal, then a = b, b = c, c = d, and d = a, which means a = b = c = d = 8. The only such quadruple then is (8,8,8,8).
If (a,b,c,d) and (b,c,d,a) are equal, then a = b, b = c, c = d, and d = a, which means a = b = c = d = 8. The only such quadruple then is (8,8,8,8).
rrusczyk
2010-02-10 20:46:10
Next, suppose the first and third quadruples (a,b,c,d) and (c,d,a,b) are equal. What does this say?
Next, suppose the first and third quadruples (a,b,c,d) and (c,d,a,b) are equal. What does this say?
ksun48
2010-02-10 20:46:28
a=c, b=d
a=c, b=d
darkdieuguerre
2010-02-10 20:46:28
a=c and b=d
a=c and b=d
pumpkinpi
2010-02-10 20:46:28
a=c, b=d
a=c, b=d
nikeballa96
2010-02-10 20:46:28
a=c, b=d
a=c, b=d
tenniskidperson3
2010-02-10 20:46:28
a=c, b=d, a+b=16
a=c, b=d, a+b=16
BarbieRocks
2010-02-10 20:46:28
a=c, b=d,
a=c, b=d,
KingJames
2010-02-10 20:46:31
a=c, b=d
a=c, b=d
rrusczyk
2010-02-10 20:46:54
OK, and how many pairs do we have here?
OK, and how many pairs do we have here?
briantix
2010-02-10 20:47:24
7
7
33286
2010-02-10 20:47:24
7?
7?
c32r17qp181cd912
2010-02-10 20:47:24
7.
7.
Just_Beginner
2010-02-10 20:47:30
7?
7?
rrusczyk
2010-02-10 20:47:32
If (a,b,c,d) and (c,d,a,b) are equal, then a = c and b = d. The 14 quadruples in this case are (1,15,1,15), (2,14,2,14), (3,13,3,14), ..., (15,1,15,1). (We do not include the quadruple (8,8,8,8), because we have already counted it.) These 14 quadruples correspond to 7 cyclic quadrilaterals.
If (a,b,c,d) and (c,d,a,b) are equal, then a = c and b = d. The 14 quadruples in this case are (1,15,1,15), (2,14,2,14), (3,13,3,14), ..., (15,1,15,1). (We do not include the quadruple (8,8,8,8), because we have already counted it.) These 14 quadruples correspond to 7 cyclic quadrilaterals.
rrusczyk
2010-02-10 20:47:56
Finally, suppose the first and fourth quadruples (a,b,c,d) and (d,a,b,c) are equal. What does this say?
Finally, suppose the first and fourth quadruples (a,b,c,d) and (d,a,b,c) are equal. What does this say?
darkdieuguerre
2010-02-10 20:48:17
a=b=c=d again
a=b=c=d again
yforman
2010-02-10 20:48:17
a=b=c=d=8 again
a=b=c=d=8 again
pumpkinpi
2010-02-10 20:48:17
a=b=c=d=8 again
a=b=c=d=8 again
jxl28
2010-02-10 20:48:17
a=b=c=d again
a=b=c=d again
nikeballa96
2010-02-10 20:48:17
a=b=c=d again?
a=b=c=d again?
Wikipedian1337
2010-02-10 20:48:17
again 8888
again 8888
ksun48
2010-02-10 20:48:17
they are all 8, same as first one
they are all 8, same as first one
trigfan
2010-02-10 20:48:17
8,8,8,8
8,8,8,8
rrusczyk
2010-02-10 20:48:19
Like the first case, the only possibility is (8,8,8,8).
Like the first case, the only possibility is (8,8,8,8).
rrusczyk
2010-02-10 20:48:28
Therefore, in all other quadruples, each quadruple (a,b,c,d) generates four different quadruples (a,b,c,d), (b,c,d,a), (c,d,a,b), and (d,a,b,c). How many such quadruples are there?
Therefore, in all other quadruples, each quadruple (a,b,c,d) generates four different quadruples (a,b,c,d), (b,c,d,a), (c,d,a,b), and (d,a,b,c). How many such quadruples are there?
33286
2010-02-10 20:49:03
2240/4
2240/4
7thsamurai
2010-02-10 20:49:03
(2255 - 15) / 4 = 560
(2255 - 15) / 4 = 560
rrusczyk
2010-02-10 20:49:09
The number of such quadruples is 2255 - 1 - 14 = 2240.
The number of such quadruples is 2255 - 1 - 14 = 2240.
rrusczyk
2010-02-10 20:49:15
Now we can divide by 4. These 2240 quadruples correspond to 2240/4 = 560 cyclic quadrilaterals.
Now we can divide by 4. These 2240 quadruples correspond to 2240/4 = 560 cyclic quadrilaterals.
rrusczyk
2010-02-10 20:49:24
So, is the answer A AGAIN?!?
So, is the answer A AGAIN?!?
darkdieuguerre
2010-02-10 20:49:32
560 + 1 + 7 = 568 giving us C
560 + 1 + 7 = 568 giving us C
yforman
2010-02-10 20:49:32
560+ the eight special ones = 568
560+ the eight special ones = 568
Aeroalon
2010-02-10 20:49:43
Plus the 8 we counted
Plus the 8 we counted
7thsamurai
2010-02-10 20:49:43
560 + 7 + 1 = 568
560 + 7 + 1 = 568
aleph0
2010-02-10 20:49:43
so then, with those 560, we add back 8,8,8,8 and the 7 rectangles to make 568 (C)
so then, with those 560, we add back 8,8,8,8 and the 7 rectangles to make 568 (C)
pirox
2010-02-10 20:49:45

rrusczyk
2010-02-10 20:50:11
Ah, yeah. Gotta remember the special cases -- we add those back in. That gives us 1+7+560 = 568 cyclic quadrilaterals...
Ah, yeah. Gotta remember the special cases -- we add those back in. That gives us 1+7+560 = 568 cyclic quadrilaterals...
rrusczyk
2010-02-10 20:50:43
I'll take another short break, and then take requests.
I'll take another short break, and then take requests.
pt90
2010-02-10 20:52:00
On number 24, sin(8x)=0 for 7 values, sin(7x)=0 for 6, sin(6x)=0 for 5, sin(5x)=0 for 4, then the rest are "repeats". 7+6+5+4=22. The answer should be roughly half so around 11. Only one choice is close. it seems like an efficient, though hardly rigorous, time saver (especially on a #24). is this legitamate?
On number 24, sin(8x)=0 for 7 values, sin(7x)=0 for 6, sin(6x)=0 for 5, sin(5x)=0 for 4, then the rest are "repeats". 7+6+5+4=22. The answer should be roughly half so around 11. Only one choice is close. it seems like an efficient, though hardly rigorous, time saver (especially on a #24). is this legitamate?
rrusczyk
2010-02-10 20:52:09
Great approach for getting a sensible guess.
Great approach for getting a sensible guess.
bunnygirl
2010-02-10 20:53:17
Can you please go over #18 on amc 10 A
Can you please go over #18 on amc 10 A
rrusczyk
2010-02-10 20:54:03
This was also #16 on the AMC 12
This was also #16 on the AMC 12
rrusczyk
2010-02-10 20:54:09

rrusczyk
2010-02-10 20:54:21
First, we focus on the difference between the numbers that Bernardo can pick and the numbers that Silvia can pick. What is this difference?
First, we focus on the difference between the numbers that Bernardo can pick and the numbers that Silvia can pick. What is this difference?
Just_Beginner
2010-02-10 20:54:32
he has a 9
he has a 9
ksun48
2010-02-10 20:54:32
the 9
the 9
simplyMathLete
2010-02-10 20:54:32
B can pick 9
B can pick 9
darkdieuguerre
2010-02-10 20:54:32
Bernarndo can pick a 9
Bernarndo can pick a 9
rohbat
2010-02-10 20:54:32
9
9
infinity1
2010-02-10 20:54:32
he can pick 9
he can pick 9
bunnygirl
2010-02-10 20:54:32
9
9
rrusczyk
2010-02-10 20:54:35
Bernardo has access to the digit 9, while Silvia does not.
Bernardo has access to the digit 9, while Silvia does not.
rrusczyk
2010-02-10 20:54:40
What happens if Bernardo picks the digit 9?
What happens if Bernardo picks the digit 9?
pt90
2010-02-10 20:54:45
If he picks a 9, then he "wins". 1/3 of the time
If he picks a 9, then he "wins". 1/3 of the time
Aeroalon
2010-02-10 20:54:45
The 9. Bernardo's number is automactially higher if he picks the 9.
The 9. Bernardo's number is automactially higher if he picks the 9.
7thsamurai
2010-02-10 20:54:45
He wins
He wins
rrusczyk
2010-02-10 20:54:49
If Bernardo picks the digit 9, then his 3-digit number will start with the digit 9, so no matter what his other digits are, his number will be greater than Silvia's number. What is the probability that Bernardo picks the digit 9?
If Bernardo picks the digit 9, then his 3-digit number will start with the digit 9, so no matter what his other digits are, his number will be greater than Silvia's number. What is the probability that Bernardo picks the digit 9?
darkdieuguerre
2010-02-10 20:55:02
1/3
1/3
ksun48
2010-02-10 20:55:02
the probability is 1/3
the probability is 1/3
infinity1
2010-02-10 20:55:02
1/3
1/3
dday25
2010-02-10 20:55:02
1/3
1/3
happymath
2010-02-10 20:55:02
1/3
1/3
nikeballa96
2010-02-10 20:55:02
1/3
1/3
rrusczyk
2010-02-10 20:55:06
The probability that Bernardo picks the digit 9 is simply 3/9 = 1/3, because he picks 3 digits out of 9.
The probability that Bernardo picks the digit 9 is simply 3/9 = 1/3, because he picks 3 digits out of 9.
rrusczyk
2010-02-10 20:55:16
If Bernardo does not pick the digit 9, then Bernardo and Silvia have access to the same set of digits. Does that mean the probability that Bernardo's number is greater than Silvia's is 1/2?
If Bernardo does not pick the digit 9, then Bernardo and Silvia have access to the same set of digits. Does that mean the probability that Bernardo's number is greater than Silvia's is 1/2?
challenger
2010-02-10 20:55:30
nope
nope
nikeballa96
2010-02-10 20:55:30
no. they can be the same.
no. they can be the same.
darkdieuguerre
2010-02-10 20:55:30
no, they could pick equal numbers
no, they could pick equal numbers
jxl28
2010-02-10 20:55:30
No, they can be equal
No, they can be equal
smokewisps
2010-02-10 20:55:30
no, not if they're equal
no, not if they're equal
ksun48
2010-02-10 20:55:30
no equals is not counted for
no equals is not counted for
rrusczyk
2010-02-10 20:55:35
No, because Bernardo's number could be equal to Silvia's. What is the probability that this occurs?
No, because Bernardo's number could be equal to Silvia's. What is the probability that this occurs?
pt90
2010-02-10 20:55:48
1/56 chance it will be the same number
1/56 chance it will be the same number
dday25
2010-02-10 20:55:48
1/56
1/56
megahertz
2010-02-10 20:55:48
1/56
1/56
ytrewq
2010-02-10 20:55:48
1/56
1/56
trigfan
2010-02-10 20:55:48
1/56
1/56
yforman
2010-02-10 20:55:53
1/(8c3) = 1/56
1/(8c3) = 1/56
rrusczyk
2010-02-10 20:55:57

rrusczyk
2010-02-10 20:56:07
So given that Bernardo does not pick the digit 9, what is the probability that Bernardo's number is greater than Silvia's?
So given that Bernardo does not pick the digit 9, what is the probability that Bernardo's number is greater than Silvia's?
nikeballa96
2010-02-10 20:56:32
the rest of it will be split evenly between silvia and bernardo
the rest of it will be split evenly between silvia and bernardo
darkdieuguerre
2010-02-10 20:56:32
1/2(1-1/56)
1/2(1-1/56)
ytrewq
2010-02-10 20:56:32
1/2*55/56
1/2*55/56
simplyMathLete
2010-02-10 20:56:32
55/112
55/112
gh625
2010-02-10 20:56:32
55/112
55/112
megahertz
2010-02-10 20:56:32
1/2(1- 1/56)=55/112
1/2(1- 1/56)=55/112
dday25
2010-02-10 20:56:32
1/2(1-1/56)
1/2(1-1/56)
rrusczyk
2010-02-10 20:56:36
The probability that Bernardo and Silvia do not pick the same set of digits is 1 - 1/56 = 55/56. Then by symmetry, the probability that Bernardo's number is greater than Silvia's is simply half of this, or 55/112.
The probability that Bernardo and Silvia do not pick the same set of digits is 1 - 1/56 = 55/56. Then by symmetry, the probability that Bernardo's number is greater than Silvia's is simply half of this, or 55/112.
rrusczyk
2010-02-10 20:56:40
And how do we finish?
And how do we finish?
rrusczyk
2010-02-10 20:57:05
Careful!! I see some of you adding this to 1/3
Careful!! I see some of you adding this to 1/3
pt90
2010-02-10 20:57:21
so 1/2*55/56 for the case where he doesn't pick a 9, which happens two thirds of the time. so this times 2/3 plus 1/3 is the answer
so 1/2*55/56 for the case where he doesn't pick a 9, which happens two thirds of the time. so this times 2/3 plus 1/3 is the answer
ytrewq
2010-02-10 20:57:21
2/3 of that plus 1/3=37/56
2/3 of that plus 1/3=37/56
challenger
2010-02-10 20:57:21
55/112*2/3 + 1/3
55/112*2/3 + 1/3
trigfan
2010-02-10 20:57:21
2/3*55/112+1/3=37/56
2/3*55/112+1/3=37/56
rrusczyk
2010-02-10 20:57:26

rrusczyk
2010-02-10 20:57:38
That 55/112 is only the probably 2/3 of the time.
That 55/112 is only the probably 2/3 of the time.
rrusczyk
2010-02-10 20:58:02
I'll do #17 on the AMC 12, which is #19 on the AMC 10
I'll do #17 on the AMC 12, which is #19 on the AMC 10
rrusczyk
2010-02-10 20:58:22

nikeballa96
2010-02-10 20:58:45
draw a picture.
draw a picture.
rrusczyk
2010-02-10 20:58:49

rrusczyk
2010-02-10 20:58:56
Since hexagon ABCDEF is equiangular, all of its angles are equal to 120 degrees. So what does this say about triangle ACE?
Since hexagon ABCDEF is equiangular, all of its angles are equal to 120 degrees. So what does this say about triangle ACE?
darkdieuguerre
2010-02-10 20:59:12
equilateral
equilateral
mz94
2010-02-10 20:59:12
equilateral
equilateral
PowerOfPi
2010-02-10 20:59:12
equilateral
equilateral
professordad
2010-02-10 20:59:12
equilateral
equilateral
ksun48
2010-02-10 20:59:12
equilateral
equilateral
RunpengFAILS
2010-02-10 20:59:12
IT IS ALSO EQUILATERAL
IT IS ALSO EQUILATERAL
pianoforte
2010-02-10 20:59:17
equilateral from symmetry
equilateral from symmetry
rrusczyk
2010-02-10 20:59:21
By SAS, triangles ABC, CDE, and EFA are all congruent. Therefore, AC = CE = EA, which means triangle ACE is equilateral.
By SAS, triangles ABC, CDE, and EFA are all congruent. Therefore, AC = CE = EA, which means triangle ACE is equilateral.
rrusczyk
2010-02-10 20:59:29
How can we find the area of equilateral triangle ACE?
How can we find the area of equilateral triangle ACE?
nikeballa96
2010-02-10 20:59:45
cosine law on DCE to find CE
cosine law on DCE to find CE
joeislittle
2010-02-10 20:59:45
we can determine its area using cosines
we can determine its area using cosines
kp54
2010-02-10 20:59:45
law of cosines
law of cosines
darkdieuguerre
2010-02-10 20:59:48
find the side length
find the side length
yforman
2010-02-10 20:59:50
use law of cosines to find the length of each side
use law of cosines to find the length of each side
rrusczyk
2010-02-10 21:00:02
We can apply the law of cosines.
We can apply the law of cosines.
rrusczyk
2010-02-10 21:00:11

rrusczyk
2010-02-10 21:00:47
(For those of you who don't know the Law of Cosines, there is a clever solution that involves extending the long sides of the hexagon to form an equilateral triangle.)
(For those of you who don't know the Law of Cosines, there is a clever solution that involves extending the long sides of the hexagon to form an equilateral triangle.)
mz94
2010-02-10 21:00:59
what if we drop a perpendicular from C to AB at X. since we know CXB is a 30 60 90 triangle we have BX=r/2 and CX=r(sqrt3)/2 adn then use pythag
what if we drop a perpendicular from C to AB at X. since we know CXB is a 30 60 90 triangle we have BX=r/2 and CX=r(sqrt3)/2 adn then use pythag
rrusczyk
2010-02-10 21:01:11
It involves cleverly dropping an altitude like this.
It involves cleverly dropping an altitude like this.
rrusczyk
2010-02-10 21:01:20
So, we have AC^2
So, we have AC^2
rrusczyk
2010-02-10 21:01:25
What do we do with that?
What do we do with that?
BarbieRocks
2010-02-10 21:01:39
i drew FC and found the total area using the two trapezoids, becasue FC=r+1
i drew FC and found the total area using the two trapezoids, becasue FC=r+1
rrusczyk
2010-02-10 21:01:45
That's pretty clever, too.
That's pretty clever, too.
joeislittle
2010-02-10 21:01:52
find the area of the triangle in terms of r
find the area of the triangle in terms of r
7thsamurai
2010-02-10 21:01:53
We can find the area of ACE with it: AC^2 sqrt(3) / 4
We can find the area of ACE with it: AC^2 sqrt(3) / 4
rrusczyk
2010-02-10 21:02:01

simplyMathLete
2010-02-10 21:02:28
and then we could use absinC/2 and complemetary counting
and then we could use absinC/2 and complemetary counting
Aeroalon
2010-02-10 21:02:35
We have been told this is 70% of the hexagon
We have been told this is 70% of the hexagon
lxu1
2010-02-10 21:02:35
we can find the area of the rest using 1/2sine120xr
we can find the area of the rest using 1/2sine120xr
rrusczyk
2010-02-10 21:02:40
We are given that the area of triangle ACE is 70% of the area of the hexagon.
We are given that the area of triangle ACE is 70% of the area of the hexagon.
rrusczyk
2010-02-10 21:02:47
The areas of triangles ABC, CDE, and EFA then collectively make up the remaining 30% of the area of the hexagon.
The areas of triangles ABC, CDE, and EFA then collectively make up the remaining 30% of the area of the hexagon.
rrusczyk
2010-02-10 21:02:51
Furthermore, these three triangles are congruent, so the area of each such triangle is 10% of the area of the hexagon. What is the area of triangle ABC in terms of r?
Furthermore, these three triangles are congruent, so the area of each such triangle is 10% of the area of the hexagon. What is the area of triangle ABC in terms of r?
pumpkinpi
2010-02-10 21:03:39
r(sqrt3)(1/4)
r(sqrt3)(1/4)
PowerOfPi
2010-02-10 21:03:39
r*sqrt(3)/4
r*sqrt(3)/4
lxu1
2010-02-10 21:03:39
sqrt3/4 x r
sqrt3/4 x r
7thsamurai
2010-02-10 21:03:39
1 * r * sin 120 / 2 = r sqrt(3) / 4. I think.
1 * r * sin 120 / 2 = r sqrt(3) / 4. I think.
rrusczyk
2010-02-10 21:03:45

rrusczyk
2010-02-10 21:03:55
All right, what equation does this give us?
All right, what equation does this give us?
darkdieuguerre
2010-02-10 21:04:34
(r*sqrt(3)/4) / (r^2+r+1 * sqrt(3) / 4) = 1/7
(r*sqrt(3)/4) / (r^2+r+1 * sqrt(3) / 4) = 1/7
33286
2010-02-10 21:04:38
r^2+r+1=7r
r^2+r+1=7r
Aeroalon
2010-02-10 21:04:38
7r = r^2 + r + 1
7r = r^2 + r + 1
rrusczyk
2010-02-10 21:04:42

rrusczyk
2010-02-10 21:04:54
What is the sum of all possible values of r?
What is the sum of all possible values of r?
nikeballa96
2010-02-10 21:05:00
hence, the sum is 6, by vietas!
hence, the sum is 6, by vietas!
professordad
2010-02-10 21:05:00
so the sum is -(-6) is 6!
so the sum is -(-6) is 6!
darkdieuguerre
2010-02-10 21:05:00
by vieta's, the sum of the roots is 6
by vieta's, the sum of the roots is 6
pumpkinpi
2010-02-10 21:05:05
By Vieta, the sum of the roots is 6
By Vieta, the sum of the roots is 6
7thsamurai
2010-02-10 21:05:05
6
6
jxl28
2010-02-10 21:05:09
6.
6.
EricMathPath09
2010-02-10 21:05:11
6
6
gh625
2010-02-10 21:05:11
6
6
rrusczyk
2010-02-10 21:05:13
By Vieta's formulas for the sum and product of the roots of a quadratic, the sum of all possible values of r is 6. The answer is (E). (For good measure, we check that the roots of the quadratic, namely 3 +/- 2 sqrt(2), are real.)
By Vieta's formulas for the sum and product of the roots of a quadratic, the sum of all possible values of r is 6. The answer is (E). (For good measure, we check that the roots of the quadratic, namely 3 +/- 2 sqrt(2), are real.)
mz94
2010-02-10 21:05:16
another way to find the area of the hexagon is to extend AB, CD, and EF to form a equilateral triangle with side length r+2
another way to find the area of the hexagon is to extend AB, CD, and EF to form a equilateral triangle with side length r+2
rrusczyk
2010-02-10 21:05:24
Yep, that would work, too.
Yep, that would work, too.
rrusczyk
2010-02-10 21:05:50
#18 AMC 12
#18 AMC 12
rrusczyk
2010-02-10 21:05:51

rrusczyk
2010-02-10 21:06:17
We'll take a look at a couple different ways to do this.
We'll take a look at a couple different ways to do this.
rrusczyk
2010-02-10 21:06:26
Any ideas?
Any ideas?
yankeesrule007
2010-02-10 21:06:35
recursion?
recursion?
rrusczyk
2010-02-10 21:06:44
Let's try something a little like this:
Let's try something a little like this:
mathrat
2010-02-10 21:06:47
draw out the paths diagram- (like pascals triangle)
draw out the paths diagram- (like pascals triangle)
dday25
2010-02-10 21:06:49
Counting along gridlines.
Counting along gridlines.
rrusczyk
2010-02-10 21:07:03
If we just started at (-4,-4), how many ways can we get to (-4,-4)?
If we just started at (-4,-4), how many ways can we get to (-4,-4)?
Aeroalon
2010-02-10 21:07:11
1
1
aerrowfinn72
2010-02-10 21:07:11
1
1
lasse
2010-02-10 21:07:11
1
1
pumpkinpi
2010-02-10 21:07:14
1: stay
1: stay
Rocket95
2010-02-10 21:07:14
1
1
Canis Lupus
2010-02-10 21:07:14
1
1
yankeesrule007
2010-02-10 21:07:14
1
1
rrusczyk
2010-02-10 21:07:16
Um, 1.
Um, 1.
rrusczyk
2010-02-10 21:07:30
How about from (-3,-4) or from (-4,-3)?
How about from (-3,-4) or from (-4,-3)?
pt90
2010-02-10 21:07:41
divide the problem into halves since it's symmetric
divide the problem into halves since it's symmetric
yankeesrule007
2010-02-10 21:07:41
1
1
Aeroalon
2010-02-10 21:07:41
Also 1
Also 1
Rocket95
2010-02-10 21:07:41
1
1
lasse
2010-02-10 21:07:45
1
1
darkdieuguerre
2010-02-10 21:07:45
1, going either up or right
1, going either up or right
rrusczyk
2010-02-10 21:07:52
There is only one way to reach the point (-3,-4). Similarly, there is only one way to reach the point (-4,-3), so we label these points with the number 1.
There is only one way to reach the point (-3,-4). Similarly, there is only one way to reach the point (-4,-3), so we label these points with the number 1.
rrusczyk
2010-02-10 21:07:57

rrusczyk
2010-02-10 21:08:03
How many ways can we reach the point (-3,-3)?
How many ways can we reach the point (-3,-3)?
rrusczyk
2010-02-10 21:08:17
That is, how many ways can we get to (-4,-4) from (-3,-3)
That is, how many ways can we get to (-4,-4) from (-3,-3)
andrewnycpops
2010-02-10 21:08:21
2
2
Aeroalon
2010-02-10 21:08:21
2: 1 + 1
2: 1 + 1
utsamagoots3
2010-02-10 21:08:21
2
2
pumpkinpi
2010-02-10 21:08:21
2
2
zigzag
2010-02-10 21:08:21
2
2
etude
2010-02-10 21:08:21
2
2
megahertz
2010-02-10 21:08:21
2
2
7thsamurai
2010-02-10 21:08:21
2; add the 1's
2; add the 1's
trigfan
2010-02-10 21:08:21
2
2
karanbir
2010-02-10 21:08:21
2
2
rrusczyk
2010-02-10 21:08:37
There are only two ways to reach (-4,-4) from the point (-3,-3), so we label this point with the number 2. This is the sum of labels at the points (-3,-4) and (-4,-3), because these are the only points through which to reach the point (-4,-4).
There are only two ways to reach (-4,-4) from the point (-3,-3), so we label this point with the number 2. This is the sum of labels at the points (-3,-4) and (-4,-3), because these are the only points through which to reach the point (-4,-4).
rrusczyk
2010-02-10 21:08:42

pirox
2010-02-10 21:08:46
keep going till you get to 4,4
keep going till you get to 4,4
simplyMathLete
2010-02-10 21:08:46
the entire bottom row is 1s, and all the left column is 1s
the entire bottom row is 1s, and all the left column is 1s
rrusczyk
2010-02-10 21:09:04
Exactly -- we just keep doing this reasoning all the way back to (4,4)
Exactly -- we just keep doing this reasoning all the way back to (4,4)
pumpkinpi
2010-02-10 21:09:08
it's looking a lot like pascal's triangle
it's looking a lot like pascal's triangle
Chaos
2010-02-10 21:09:08
rotated pascal's
rotated pascal's
aunch
2010-02-10 21:09:08
complete the square by adding the numbers to the left and bottom of the needed vertex
complete the square by adding the numbers to the left and bottom of the needed vertex
rrusczyk
2010-02-10 21:09:10
In general, the label at the point (x,y) will be the sum of the labels at (x - 1, y) and (x, y - 1). We just have to remember to stay outside or on the boundary of the square described by -2 <= x, y <= 2, and inside the square described by -4 <= x, y <= 4. (Without these restrictions, we would in fact generate Pascal's triangle.)
In general, the label at the point (x,y) will be the sum of the labels at (x - 1, y) and (x, y - 1). We just have to remember to stay outside or on the boundary of the square described by -2 <= x, y <= 2, and inside the square described by -4 <= x, y <= 4. (Without these restrictions, we would in fact generate Pascal's triangle.)
rrusczyk
2010-02-10 21:09:16
The rest of the diagram is completed as follows.
The rest of the diagram is completed as follows.
rrusczyk
2010-02-10 21:09:20

rrusczyk
2010-02-10 21:09:30
Hence, the answer is (D) 1698.
Hence, the answer is (D) 1698.
rrusczyk
2010-02-10 21:09:47
Now, if you didn't trust your repeated addition, there's a cleverer approach.
Now, if you didn't trust your repeated addition, there's a cleverer approach.
kp54
2010-02-10 21:10:05
paths must pass certain points namely (-4, 4), (-3, 3), (-2, 2), (4, -4), (3, -3), (2, -2),
paths must pass certain points namely (-4, 4), (-3, 3), (-2, 2), (4, -4), (3, -3), (2, -2),
rrusczyk
2010-02-10 21:10:09
The path from (-4,-4) must go through exactly one of (4,-4), (3,-3), (2,-2), (-2,2), (-3,3), and (-4,4).
The path from (-4,-4) must go through exactly one of (4,-4), (3,-3), (2,-2), (-2,2), (-3,3), and (-4,4).
rrusczyk
2010-02-10 21:10:13
There's only one path through (4,-4) and one through (-4,4). What about through (3,-3)?
There's only one path through (4,-4) and one through (-4,4). What about through (3,-3)?
yforman
2010-02-10 21:10:35
(8c1)^2 = 64
(8c1)^2 = 64
darkdieuguerre
2010-02-10 21:10:36
8^2 = 64
8^2 = 64
rrusczyk
2010-02-10 21:10:43
There are C(8,1) ways to get to (3,-3), since there are 8 steps from (-4,-4) to (3,-3), and only one is up. Similarly, to get from (3,-3) to (4,4), there are 8 steps, of which 1 is to the right. So, there are C(8,1) ways to finish the path. This gives us a total of C(8,1)*C(8,1) paths through (3,-3). Similarly, there are C(8,1)*C(8,1) paths through (-3,3).
There are C(8,1) ways to get to (3,-3), since there are 8 steps from (-4,-4) to (3,-3), and only one is up. Similarly, to get from (3,-3) to (4,4), there are 8 steps, of which 1 is to the right. So, there are C(8,1) ways to finish the path. This gives us a total of C(8,1)*C(8,1) paths through (3,-3). Similarly, there are C(8,1)*C(8,1) paths through (-3,3).
rrusczyk
2010-02-10 21:10:53
What about through (2,-2)?
What about through (2,-2)?
darkdieuguerre
2010-02-10 21:11:04
(8C2)^2 = 784
(8C2)^2 = 784
lasse
2010-02-10 21:11:04
C(8,2)*^2
C(8,2)*^2
dday25
2010-02-10 21:11:10
(8c2)^2
(8c2)^2
rrusczyk
2010-02-10 21:11:12
Similarly, there are C(8,2) paths to (2,-2), and C(8,2) paths from (2,-2) to (4,4), so there are C(8,2)*C(8,2) paths from (-4,-4) to (4,4) through (2,-2). Same goes for (-2,2).
Similarly, there are C(8,2) paths to (2,-2), and C(8,2) paths from (2,-2) to (4,4), so there are C(8,2)*C(8,2) paths from (-4,-4) to (4,4) through (2,-2). Same goes for (-2,2).
rrusczyk
2010-02-10 21:11:17
This gives us our total count: 2(C(8,0)*C(8,0) + C(8,1)*C(8,1) + C(8,2)*C(8,2)).
This gives us our total count: 2(C(8,0)*C(8,0) + C(8,1)*C(8,1) + C(8,2)*C(8,2)).
rrusczyk
2010-02-10 21:11:22
I like to do counting problems two ways to make sure I haven't overlooked something!
I like to do counting problems two ways to make sure I haven't overlooked something!
rrusczyk
2010-02-10 21:11:41
So now we're sure that we're correct!
So now we're sure that we're correct!
joeislittle
2010-02-10 21:12:02
what if we missed something twice?
what if we missed something twice?
rrusczyk
2010-02-10 21:12:12
Then we're sad :(
Then we're sad :(
ksun48
2010-02-10 21:12:33
can we do #14 on the amc 10 please? pretty please with a :rose: on top?
can we do #14 on the amc 10 please? pretty please with a :rose: on top?
rrusczyk
2010-02-10 21:12:47
I don't have a diagram handy, so you'll have to draw one on your own.
I don't have a diagram handy, so you'll have to draw one on your own.
rrusczyk
2010-02-10 21:14:01

rrusczyk
2010-02-10 21:14:58
Now, hmmm. What do we do?
Now, hmmm. What do we do?
gh625
2010-02-10 21:15:30
You know that AFC = 120
You know that AFC = 120
rrusczyk
2010-02-10 21:15:44
OK, since CFE is equilateral, we have <AFC = <DFE = 120.
OK, since CFE is equilateral, we have <AFC = <DFE = 120.
rrusczyk
2010-02-10 21:16:01
Then what?
Then what?
mathwiz314
2010-02-10 21:16:08
BAC = 60
BAC = 60
rrusczyk
2010-02-10 21:16:10
Why?
Why?
yankeesrule007
2010-02-10 21:16:44
do some angle chasing to get the angles of ABC in terms of angleBAE
do some angle chasing to get the angles of ABC in terms of angleBAE
yankeesrule007
2010-02-10 21:16:45
ACB=60+x and ABC=60-x also
ACB=60+x and ABC=60-x also
rrusczyk
2010-02-10 21:17:06
We assign a variable to the equal angles: x = <BAE = <ACD
We assign a variable to the equal angles: x = <BAE = <ACD
rrusczyk
2010-02-10 21:17:18
Then we can find other angles in terms of x.
Then we can find other angles in terms of x.
rrusczyk
2010-02-10 21:17:35
We already have <BAF = x. What's <CAE in terms of x?
We already have <BAF = x. What's <CAE in terms of x?
pumpkinpi
2010-02-10 21:18:13
60-x
60-x
jxl28
2010-02-10 21:18:13
60-x
60-x
happyof
2010-02-10 21:18:13
60-x
60-x
rrusczyk
2010-02-10 21:18:36
From triangle AFC, we have <CAE = 180 - <AFC - <ACF = 60 - x.
From triangle AFC, we have <CAE = 180 - <AFC - <ACF = 60 - x.
rrusczyk
2010-02-10 21:18:47
So, what does this give us?
So, what does this give us?
lasse
2010-02-10 21:19:11
CAB=60-x+x=60
CAB=60-x+x=60
jxl28
2010-02-10 21:19:11
So <CAE+<BAE=60-x+x=<BAC=60
So <CAE+<BAE=60-x+x=<BAC=60
pumpkinpi
2010-02-10 21:19:11
since BAE = x, CAB = BAE + CAE = x + 60 - x = 60
since BAE = x, CAB = BAE + CAE = x + 60 - x = 60
rrusczyk
2010-02-10 21:19:25
That gives us <CAB = x + (60-x) = 60.
That gives us <CAB = x + (60-x) = 60.
rrusczyk
2010-02-10 21:19:30
What does that give us?
What does that give us?
lasse
2010-02-10 21:19:41
AC:AB = 1:2 so its a 30-60-90
AC:AB = 1:2 so its a 30-60-90
jxl28
2010-02-10 21:19:41
ABC is a 30-60-90 triangle!
ABC is a 30-60-90 triangle!
mathwiz314
2010-02-10 21:19:41
ABC= 30-60-90, then ACB=90
ABC= 30-60-90, then ACB=90
PowerOfPi
2010-02-10 21:19:45
30-60-90 triangle
30-60-90 triangle
rrusczyk
2010-02-10 21:20:11
Exactly; since AB = 2AC and the angle between them is 60, this triangle is similar to a 30-60-90 triangle.
Exactly; since AB = 2AC and the angle between them is 60, this triangle is similar to a 30-60-90 triangle.
yankeesrule007
2010-02-10 21:20:28
so the answer is 90
so the answer is 90
rrusczyk
2010-02-10 21:20:30
We conclude that <ACB is 90.
We conclude that <ACB is 90.
rrusczyk
2010-02-10 21:20:59
I have time for two more problems, I'll do one from each test.
I have time for two more problems, I'll do one from each test.
rrusczyk
2010-02-10 21:21:07
Now, vote for the AMC 10 problem.
Now, vote for the AMC 10 problem.
rrusczyk
2010-02-10 21:21:45

rrusczyk
2010-02-10 21:22:00
(I'll do #20, too -- it's quick :)
(I'll do #20, too -- it's quick :)
lasse
2010-02-10 21:22:11
Total vol = 3^3=27
Total vol = 3^3=27
joshim5
2010-02-10 21:22:21
total vllume is 27
total vllume is 27
rrusczyk
2010-02-10 21:22:29
Then what?
Then what?
ss5188
2010-02-10 21:22:40
find the area of each hole and subtract overcounts
find the area of each hole and subtract overcounts
pumpkinpi
2010-02-10 21:22:41
try just making one hole and finding the volume that is taken out
try just making one hole and finding the volume that is taken out
henrypickle
2010-02-10 21:22:41
find empty space and subtract
find empty space and subtract
bulutcocuk
2010-02-10 21:22:43
subtract the holes
subtract the holes
rrusczyk
2010-02-10 21:22:45
We could start by trying to figure out what's left, but perhaps an easier strategy is to note that the initial cube has volume 3^3 = 27, and try to figure out what's cut out.
We could start by trying to figure out what's left, but perhaps an easier strategy is to note that the initial cube has volume 3^3 = 27, and try to figure out what's cut out.
rrusczyk
2010-02-10 21:22:50
What's cut out?
What's cut out?
joshim5
2010-02-10 21:23:00
the three holes are - (2*2*3)*3
the three holes are - (2*2*3)*3
aerrowfinn72
2010-02-10 21:23:00
take away three of 2*2*3..but ya needa add some back
take away three of 2*2*3..but ya needa add some back
rrusczyk
2010-02-10 21:23:18
Each hole has volume 2*2*3 = 12, for a total of 3*12 = 36 cut out.
Each hole has volume 2*2*3 = 12, for a total of 3*12 = 36 cut out.
rrusczyk
2010-02-10 21:23:27
That leaves us with 27 - 36 = -9.
That leaves us with 27 - 36 = -9.
rrusczyk
2010-02-10 21:23:28
Um.
Um.
stargroup
2010-02-10 21:23:40
It overlaps 3 times so you need to account for the overlap twice
It overlaps 3 times so you need to account for the overlap twice
aerrowfinn72
2010-02-10 21:23:40
add back 2 of 2*2*2
add back 2 of 2*2*2
ksun48
2010-02-10 21:23:40
add 2 2*2*2's
add 2 2*2*2's
ytrewq
2010-02-10 21:23:40
the center 2x2x2 is subtracted 3 times
the center 2x2x2 is subtracted 3 times
MathTwo
2010-02-10 21:23:48
add some back you counted the 2x2x2 cube 3 times
add some back you counted the 2x2x2 cube 3 times
jxl28
2010-02-10 21:23:50
You need to add back (2*2*2)*2
You need to add back (2*2*2)*2
rrusczyk
2010-02-10 21:24:05
We subtracted the central overlap (a 2x2x2 cube) of the holes three times.
We subtracted the central overlap (a 2x2x2 cube) of the holes three times.
rrusczyk
2010-02-10 21:24:11
We add it back twice.
We add it back twice.
pirox
2010-02-10 21:24:19
-9+2*8=7
-9+2*8=7
henrypickle
2010-02-10 21:24:19
which gives -9+16=7
which gives -9+16=7
myk
2010-02-10 21:24:19
the center cube overlaps 3 times, add 2(2^3) to -9 to get 7
the center cube overlaps 3 times, add 2(2^3) to -9 to get 7
rrusczyk
2010-02-10 21:24:32

rrusczyk
2010-02-10 21:24:43
We want to make the longest possible path. Let's start simple. What's the longest path the fly could take with the first step?
We want to make the longest possible path. Let's start simple. What's the longest path the fly could take with the first step?
dolk
2010-02-10 21:25:30
sqrt(3)
sqrt(3)
7thsamurai
2010-02-10 21:25:30
sqrt(3)?
sqrt(3)?
bobma99
2010-02-10 21:25:30
the opposite corner of the box
the opposite corner of the box
darkdieuguerre
2010-02-10 21:25:30
\sqrt{3} to the diagonally opposite corner
\sqrt{3} to the diagonally opposite corner
karanbir
2010-02-10 21:25:30
diagonagly opposite flying
diagonagly opposite flying
GoldenFrog1618
2010-02-10 21:25:33
opposite corner
opposite corner
PowerOfPi
2010-02-10 21:25:33
space diagonal = sqrt(3)
space diagonal = sqrt(3)
rrusczyk
2010-02-10 21:25:40
The longest first step would be an interior diagonal, which has length sqrt(3).
The longest first step would be an interior diagonal, which has length sqrt(3).
rrusczyk
2010-02-10 21:25:44
So, there are 8 steps. Why wouldn't the answer just be 8*sqrt(3)?
So, there are 8 steps. Why wouldn't the answer just be 8*sqrt(3)?
bobma99
2010-02-10 21:26:05
you cant go back and forth 8 times
you cant go back and forth 8 times
dday25
2010-02-10 21:26:05
Only 4 such diagonals.
Only 4 such diagonals.
dnkywin
2010-02-10 21:26:05
there are only4 diagonals
there are only4 diagonals
pumpkinpi
2010-02-10 21:26:05
there are only 4 interior diagonals
there are only 4 interior diagonals
henrypickle
2010-02-10 21:26:08
8*sqrt 3 would be traveling back and forth between the two corners
8*sqrt 3 would be traveling back and forth between the two corners
rrusczyk
2010-02-10 21:26:14
The fly won't ever cover the same interior diagonal twice, and there are 4 interior diagonals. So, at most 4 of the steps could be along interior diagonals. These four together have length 4sqrt(3).
The fly won't ever cover the same interior diagonal twice, and there are 4 interior diagonals. So, at most 4 of the steps could be along interior diagonals. These four together have length 4sqrt(3).
rrusczyk
2010-02-10 21:26:18
What is the longest the other 4 paths could be?
What is the longest the other 4 paths could be?
pirox
2010-02-10 21:26:27

pumpkinpi
2010-02-10 21:26:27
sqrt2
sqrt2
moplam
2010-02-10 21:26:27
sqrt 2
sqrt 2
jxl28
2010-02-10 21:26:27
sqrt(2)
sqrt(2)
ksun48
2010-02-10 21:26:27
\sqrt2
\sqrt2
bulutcocuk
2010-02-10 21:26:27
root(2)
root(2)
rpond
2010-02-10 21:26:27
sqrt(2)
sqrt(2)
megahertz
2010-02-10 21:26:27
sqrt{2}
sqrt{2}
gh625
2010-02-10 21:26:27
sqrt{2}
sqrt{2}
LincolnMS nerd herd
2010-02-10 21:26:27

rrusczyk
2010-02-10 21:26:32
The longest the other 4 paths could be are face diagonals. This would add another 4sqrt(2) to the total length of the path.
The longest the other 4 paths could be are face diagonals. This would add another 4sqrt(2) to the total length of the path.
rrusczyk
2010-02-10 21:26:36
What's left to check?
What's left to check?
ksun48
2010-02-10 21:26:46
if it is possible
if it is possible
turak
2010-02-10 21:26:46
if its possible
if its possible
joeislittle
2010-02-10 21:26:46
this is possible
this is possible
moplam
2010-02-10 21:26:46
is it possible
is it possible
pumpkinpi
2010-02-10 21:26:46
can this be accomplished?
can this be accomplished?
darkdieuguerre
2010-02-10 21:26:49
that it's actually possible
that it's actually possible
7thsamurai
2010-02-10 21:26:49
That this is possible.
That this is possible.
dday25
2010-02-10 21:26:49
If this is all possible in one path.
If this is all possible in one path.
jenniferyin
2010-02-10 21:26:49
if its possible
if its possible
rrusczyk
2010-02-10 21:26:54
We must check if such a path is possible -- can the fly visit all the vertices with four space diagonals and four face diagonals?
We must check if such a path is possible -- can the fly visit all the vertices with four space diagonals and four face diagonals?
ksun48
2010-02-10 21:27:17
yes
yes
vv123
2010-02-10 21:27:17
alternate rt3 and rt2 and get 4rt3+4rt2
alternate rt3 and rt2 and get 4rt3+4rt2
happyof
2010-02-10 21:27:17
yes
yes
challenger
2010-02-10 21:27:17
yep
yep
soulspeedy
2010-02-10 21:27:17
yep
yep
dday25
2010-02-10 21:27:21
Yes, it can if it alternates between the two types of diagonals.
Yes, it can if it alternates between the two types of diagonals.
rrusczyk
2010-02-10 21:27:25
Yes. It can do so with a sequence of face-space diagonal pairs.
Yes. It can do so with a sequence of face-space diagonal pairs.
rrusczyk
2010-02-10 21:27:30

rrusczyk
2010-02-10 21:27:38
That's the first pair.
That's the first pair.
rrusczyk
2010-02-10 21:27:40

rrusczyk
2010-02-10 21:27:48
There are the rest.
There are the rest.
rrusczyk
2010-02-10 21:27:59
Start anywhere :)
Start anywhere :)
pirox
2010-02-10 21:28:07
so it's (D)! :-D
so it's (D)! :-D
yankeesrule007
2010-02-10 21:28:07
so the asnwer is D
so the asnwer is D
ksun48
2010-02-10 21:28:07
red is plane, blue is space
red is plane, blue is space
Iggy Iguana
2010-02-10 21:28:07
so it is 4sqrt3 + 4 sqrt2
so it is 4sqrt3 + 4 sqrt2
GoldenRatioOfPhi
2010-02-10 21:28:07
answer ia d
answer ia d
moplam
2010-02-10 21:28:09
Nice diagram
Nice diagram
rrusczyk
2010-02-10 21:28:12
Thanks :)
Thanks :)
rrusczyk
2010-02-10 21:28:19
OK, one more AMC 12 problem.
OK, one more AMC 12 problem.
rrusczyk
2010-02-10 21:28:21
Vote time
Vote time
rrusczyk
2010-02-10 21:28:46

rrusczyk
2010-02-10 21:29:26
Let's look the first arithmetic sequence (a_n) first.
Let's look the first arithmetic sequence (a_n) first.
rrusczyk
2010-02-10 21:29:35
Let d_1 be its common difference. Then what is the nth term a_n?
Let d_1 be its common difference. Then what is the nth term a_n?
crazypianist1116
2010-02-10 21:30:09
1 +(n-1)d
1 +(n-1)d
yankeesrule007
2010-02-10 21:30:09
1+ d_1(n-1)
1+ d_1(n-1)
stargroup
2010-02-10 21:30:09
1 + (n-1)d_1
1 + (n-1)d_1
yforman
2010-02-10 21:30:09
1+(n-1)d_1
1+(n-1)d_1
mgao
2010-02-10 21:30:09
a_n = d_1(n-1)+1
a_n = d_1(n-1)+1
tonypr
2010-02-10 21:30:09

rrusczyk
2010-02-10 21:30:12

rrusczyk
2010-02-10 21:30:20

rrusczyk
2010-02-10 21:30:29

rrusczyk
2010-02-10 21:30:41
What can we do with these ?
What can we do with these ?
rrusczyk
2010-02-10 21:31:09
I see some of you suggesting multiplying them.
I see some of you suggesting multiplying them.
rrusczyk
2010-02-10 21:31:19
That's gonna create an awful mess, no?
That's gonna create an awful mess, no?
GoldenRatioOfPhi
2010-02-10 21:31:22
subtract
subtract
rrusczyk
2010-02-10 21:31:27
Subtract what?
Subtract what?
crazypianist1116
2010-02-10 21:31:32
anbn=2010
anbn=2010
MathTwo
2010-02-10 21:31:32
a_n and b_n are factors of 2010
a_n and b_n are factors of 2010
karanbir
2010-02-10 21:31:36
an(bn)=2010
an(bn)=2010
rrusczyk
2010-02-10 21:31:48
We do know that a_n and b_n multiply to 2010
We do know that a_n and b_n multiply to 2010
rrusczyk
2010-02-10 21:31:55
Can we learn more from these equations?
Can we learn more from these equations?
moplam
2010-02-10 21:31:58
1 from both side
1 from both side
rrusczyk
2010-02-10 21:32:04
Let's try that.
Let's try that.
rrusczyk
2010-02-10 21:32:05

rrusczyk
2010-02-10 21:32:13
What does this tell us?
What does this tell us?
yankeesrule007
2010-02-10 21:32:38
Find to factors of 2010 which when you subtract them from 1, share a common factor
Find to factors of 2010 which when you subtract them from 1, share a common factor
kthxbai
2010-02-10 21:32:38
a_n=b_n=1 (mod n-1)
a_n=b_n=1 (mod n-1)
karanbir
2010-02-10 21:32:45
devide by n-1
devide by n-1
rrusczyk
2010-02-10 21:32:53

rrusczyk
2010-02-10 21:33:09
And what do we know about a_n and b_n that will help?
And what do we know about a_n and b_n that will help?
megahertz
2010-02-10 21:33:22
multiplies to 2010
multiplies to 2010
rpond
2010-02-10 21:33:22
that they multiply to 2010, of course
that they multiply to 2010, of course
BarbieRocks
2010-02-10 21:33:22
they multiply to 2010!!!
they multiply to 2010!!!
rrusczyk
2010-02-10 21:33:37
All right, we know all our possibilities for a_n and b_n.
All right, we know all our possibilities for a_n and b_n.
rrusczyk
2010-02-10 21:33:44
We could go ahead and list them:
We could go ahead and list them:
rrusczyk
2010-02-10 21:33:49

rrusczyk
2010-02-10 21:34:02
As we crank through these, what are we looking for?
As we crank through these, what are we looking for?
rpond
2010-02-10 21:34:33
pairs that are both 1 (mod k) for some k
pairs that are both 1 (mod k) for some k
ShocK n AwE
2010-02-10 21:34:33
now we look for if we subtract 1, they should have a common factor besides 1
now we look for if we subtract 1, they should have a common factor besides 1
yforman
2010-02-10 21:34:33
the largest possible gcd of a_n-1 and b_n-1
the largest possible gcd of a_n-1 and b_n-1
BarbieRocks
2010-02-10 21:34:36
two numbers such that when you subtract 1 from each of them, they have a common factor
two numbers such that when you subtract 1 from each of them, they have a common factor
rrusczyk
2010-02-10 21:34:39
We're looking for the pair such that subtracting 1 from each gives a pair with a high GCD
We're looking for the pair such that subtracting 1 from each gives a pair with a high GCD
rrusczyk
2010-02-10 21:35:10

GoldenFrog1618
2010-02-10 21:35:12
(15,134) gives a n-1=7
(15,134) gives a n-1=7
ShocK n AwE
2010-02-10 21:35:13
15,134 is the only one
15,134 is the only one
rpond
2010-02-10 21:35:18
but there's only one pair that does this: (15-1, 134-1) gives us a GCD of 7
but there's only one pair that does this: (15-1, 134-1) gives us a GCD of 7
MathTwo
2010-02-10 21:35:36
then n=8
then n=8
MathTwo
2010-02-10 21:35:36
then n=8=>C
then n=8=>C
BarbieRocks
2010-02-10 21:35:36
so n=8.
so n=8.
ShocK n AwE
2010-02-10 21:35:39
n=8, yay
n=8, yay
CubeX
2010-02-10 21:35:40
SO the answer is (C)=8
SO the answer is (C)=8
rrusczyk
2010-02-10 21:35:42

rrusczyk
2010-02-10 21:35:51
Therefore, the largest possible value of n is 8, and the answer is (C)
Therefore, the largest possible value of n is 8, and the answer is (C)
rrusczyk
2010-02-10 21:36:25
That's it for the Math Jam tonight. Thank you all for coming, and special thanks to our assistant, Wendy Hou (wobster109)
That's it for the Math Jam tonight. Thank you all for coming, and special thanks to our assistant, Wendy Hou (wobster109)
aunch
2010-02-10 21:36:37
just a side question... ive noticed many problems on not only the AMCs but also other math contests that they have a lot of problems regarding the year number.... for example this year there was a lot of problems with the number 2010.... is it a good idea to take a little time to memorize some proporties of 2010 like prime factorization, remainder when divided by 7, etc.?
just a side question... ive noticed many problems on not only the AMCs but also other math contests that they have a lot of problems regarding the year number.... for example this year there was a lot of problems with the number 2010.... is it a good idea to take a little time to memorize some proporties of 2010 like prime factorization, remainder when divided by 7, etc.?
rrusczyk
2010-02-10 21:36:39
Yes.
Yes.
rrusczyk
2010-02-10 21:36:57
I see a lot of you asking about the index or average scores.
I see a lot of you asking about the index or average scores.
rrusczyk
2010-02-10 21:37:04
I really have no idea what it will be!
I really have no idea what it will be!
rrusczyk
2010-02-10 21:37:10
There's a lot of discussion on the forum about it
There's a lot of discussion on the forum about it
rrusczyk
2010-02-10 21:37:23
I also don't know how the AMC is handling the snow storms.
I also don't know how the AMC is handling the snow storms.
bobma99
2010-02-10 21:37:26
will this be saved somewhere so we can look over it again?
will this be saved somewhere so we can look over it again?
rrusczyk
2010-02-10 21:37:36
Yes, I will put the transcript up tonight in the Math Jam section
Yes, I will put the transcript up tonight in the Math Jam section
rrusczyk
2010-02-10 21:37:42
Under transcripts.
Under transcripts.
rrusczyk
2010-02-10 21:38:02
I think a lot of people found the test hard this year, so if you did, you're not alone.
I think a lot of people found the test hard this year, so if you did, you're not alone.
henrypickle
2010-02-10 21:38:37
when does the amc website usually post things like the answer key, aime qualifying scores, etc.?
when does the amc website usually post things like the answer key, aime qualifying scores, etc.?
rrusczyk
2010-02-10 21:38:57
I'm not sure -- they usually post the qualifying scores on our site before posting anywhere else.
I'm not sure -- they usually post the qualifying scores on our site before posting anywhere else.
rrusczyk
2010-02-10 21:39:13
(I don't have a guess as to whether they will be lowered or not)
(I don't have a guess as to whether they will be lowered or not)
darkdieuguerre
2010-02-10 21:39:46
an answer key for both tests is already up in the forum :)
an answer key for both tests is already up in the forum :)
rrusczyk
2010-02-10 21:40:12
Again, I really don't have a guess for what the index will be!
Again, I really don't have a guess for what the index will be!
rrusczyk
2010-02-10 21:40:25
Some of you have asked if you should take the B test if you passed the A
Some of you have asked if you should take the B test if you passed the A
rrusczyk
2010-02-10 21:40:38
Yes -- the USAMO qualification is a combination of AMC and AIME scores.
Yes -- the USAMO qualification is a combination of AMC and AIME scores.
rrusczyk
2010-02-10 21:41:39
For those of you asking about the snow, I suggest talking to the AMC
For those of you asking about the snow, I suggest talking to the AMC
Visitor03
2010-02-10 21:41:46
If I passed the AMC 10A but didn't pass the B exam, would I still qualify for AIME?
If I passed the AMC 10A but didn't pass the B exam, would I still qualify for AIME?
rrusczyk
2010-02-10 21:41:48
Yes
Yes
mathwiz314
2010-02-10 21:41:50
My school doesn't offer the 10B, but offers the 12B, should I take the 12B if I passed the 10A?
My school doesn't offer the 10B, but offers the 12B, should I take the 12B if I passed the 10A?
rrusczyk
2010-02-10 21:42:03
I generally recommend students take both if they're strong enough to pass the 10
I generally recommend students take both if they're strong enough to pass the 10
xpmath
2010-02-10 21:42:07
hm mr. rusczyk, did you take the tests for fun or did you just randomly look at the problems
hm mr. rusczyk, did you take the tests for fun or did you just randomly look at the problems
rrusczyk
2010-02-10 21:42:14
I just did the last 5 problems for fun
I just did the last 5 problems for fun
dday25
2010-02-10 21:42:42
How well did you do? ;)
How well did you do? ;)
rrusczyk
2010-02-10 21:42:56
Pretty good :) But the 90! problem was easily the hardest for me.
Pretty good :) But the 90! problem was easily the hardest for me.
westiepaw
2010-02-10 21:43:10
Which AoPS books would you especially reccomend to prep for the 10B?
Which AoPS books would you especially reccomend to prep for the 10B?
rrusczyk
2010-02-10 21:43:18
The intro series and Volume 1
The intro series and Volume 1
megahertz
2010-02-10 21:43:44
Which would you recommend for 12 and beginning AIME?
Which would you recommend for 12 and beginning AIME?
rrusczyk
2010-02-10 21:44:04
After mastering the earlier stuff, you're ready for Intermediate series and the AIME
After mastering the earlier stuff, you're ready for Intermediate series and the AIME
aopsaccount2131
2010-02-10 21:44:52
i've noticed that a lot of the AIME problems can be solved without advanced topics (beyond second-year algebra, first-year geometry, etc.) is this generally true?
i've noticed that a lot of the AIME problems can be solved without advanced topics (beyond second-year algebra, first-year geometry, etc.) is this generally true?
rrusczyk
2010-02-10 21:44:55
yes
yes
LincolnMS nerd herd
2010-02-10 21:45:38
Is this AMC 10B easier than the A?
Is this AMC 10B easier than the A?
rrusczyk
2010-02-10 21:45:43
They try to make them the same.
They try to make them the same.
pumpkinpi
2010-02-10 21:46:04
I like the comment in Volume 1 about most 3-D geometry problems being 2-D geometry problems in disguise!
I like the comment in Volume 1 about most 3-D geometry problems being 2-D geometry problems in disguise!
rrusczyk
2010-02-10 21:46:11
:)
:)
sparkle123
2010-02-10 21:47:09
Thanks SO MUCH for the math jam! this was amazing :)
Thanks SO MUCH for the math jam! this was amazing :)
rrusczyk
2010-02-10 21:47:22
Thanks for coming -- that's a wrap for tonight.
Thanks for coming -- that's a wrap for tonight.
rrusczyk
2010-02-10 21:47:37
Feel free to write me at aops@artofproblemsolving.com if you want recommendations for books or classes!
Feel free to write me at aops@artofproblemsolving.com if you want recommendations for books or classes!
twoplusthree
2010-02-10 21:47:44
thanks
thanks
RunpengFAILS
2010-02-10 21:47:44
THANK YOU
THANK YOU
yankeesrule007
2010-02-10 21:47:44
thank you!
thank you!
nearsymmetric
2010-02-10 21:47:44
Yes, thank you -- great session!
Yes, thank you -- great session!
Visitor03
2010-02-10 21:47:44
Thank you, once again.
Thank you, once again.
limac
2010-02-10 21:47:44
thank you for the math jam! :)
thank you for the math jam! :)
mathscience
2010-02-10 21:47:44
thanks a lot for the math jam!!!!:) it was amazing
thanks a lot for the math jam!!!!:) it was amazing
xpmath
2010-02-10 21:48:07
thanks
thanks
pirox
2010-02-10 21:48:07
thanks!
thanks!
turak
2010-02-10 21:48:07
thank you
thank you
Aeroalon
2010-02-10 21:48:07
Almost 3 hours :O
Almost 3 hours :O
Ihatepie
2010-02-10 21:48:07
hard test
hard test
CubeX
2010-02-10 21:48:07
thanks
thanks
aggarwal
2010-02-10 21:48:24
see you in the B jam
see you in the B jam
westiepaw
2010-02-10 21:48:24
Thanks!
Thanks!
tonypr
2010-02-10 21:48:24
Thanks for the math jam!
Thanks for the math jam!
Copyright © 2025 AoPS Incorporated. This page is copyrighted material. You can view and print this page for your own use, but you cannot share the contents of this file with others.